Está en la página 1de 40

EXAMEN DIAGNÓSTICO Se trata de un tumor de 2.

5cm, que corresponde de acuerdo a la


clasificación TNM a un T2 (tumor de más de 2 y hasta 5cm), así
CASO CLÍNICO 1
como un ganglio ipsilateral móvil, lo cual corresponde a un N1
(ganglio axilar móvil). Por ahora no hay evidencia de enfermedad
Mujer de 65 años de edad, que acude a la consulta con usted por metastásica.
presentar una tumoración no dolorosa en la mama de 3 meses de
evolución. Cuenta con antecedente de consumo de hormonales 4. La localización más frecuente de los tumores malignos de la
orales en los últimos 5 años como tratamiento para síndrome mama es:
climatérico, además de consumo de anticonceptivos orales de los Cuadrante superior interno
20 a los 35 años como método de planificación familiar. Tiene 2 Cuadrante superior externo
hijos, a los cuales no les dio lactancia materna. A la exploración Cuadrante inferior externo
física usted detecta tumoración de 2.5cm en cuadrante superior Retroareolar
derecho de la mama izquierda, con adenopatía ipsilateral de 1cm,
móvil. Sin cambios cutáneos ni secreción a través del pezón. La localización más frecuente de los tumores mamarios es el
cuadrante superior externo (50%).
1. El diagnóstico más probable de esta paciente es:
Mastopatía fibroquística 5. El factor pronóstico más importante en cáncer de mama es:
Cáncer de mama Ganglios positivos
Fibroadenoma mamario HER2 positivo
Hiperplasia ductal atípica Receptores estrogénicos negativos
Tamaño tumoral
Esta paciente tiene factores de riesgo como el uso de hormonales
orales, no haber lactado y una tumoración en cuadrante superior Todos los anteriores son factores pronósticos en cáncer de mama,
derecho, la localización más común del cáncer de mama, además sin embargo, el factor pronóstico más importante hasta ahora es la
de adenopatías, por lo que la sospecha inicial debe ser malignidad. presencia de ganglios positivos, el cual influye en la supervivencia
global (85% de supervivencia a 5 años en el grupo de ganglios
2. Su conducta inicial ante esta paciente es la siguiente: negativos versus 50% en promedio en el grupo de ganglios
Realizar escisión de la lesión positivos).
Mastografía y toma de biopsia
Ultrasonido mamario CASO CLÍNICO 2
Iniciar quimioterapia neoadyuvante
Mujer de 40 años, que acude a la consulta por presentar sangrado
Ante la sospecha de malignidad, lo indicado es realizar una transvaginal, además de flujo transvaginal fétido. Cuenta con
mastografía y tomar una biopsia para obtener el reporte antecedente de haber tenido 15 parejas sexuales, algunas de ellas
histopatológico. sin uso de métodos anticonceptivos, además de tener 3 hijos. A la
exploración física con tacto vaginal con tumoración en cérvix, de
3. De acuerdo al tamaño de la lesión, el TNM clínico es: 4cm, que involucra parametrio derecho. En los estudios de
T1N0M0 extensión se evidencia hidronefrosis derecha con función renal
T1N1M0 normal.
T2N1M0
T3N0M0 6. ¿Cuál es su diagnóstico presuntivo?
Cervicovaginitis
Cáncer cervicouterino Cirugía y radioterapia
Cáncer de vagina Quimiorradioterapia
Quiste de Naboth Quimioterapia paliativa
Solo radioterapia
Esta paciente tiene factores de riesgo para cáncer cervicouterino
como las múltiples parejas sexuales, la multiparidad. En la El tratamiento de elección en enfermedad localmente avanzada,
exploración clínica tiene una tumoración en cérvix con afección a que incluye de la etapa II B, III, IV A, es quimiorradioterapia a base
parametrio, además de hidronefrosis, por lo cual la sospecha inicial de esquemas con platinos, preferentemente cisplatino.
debe ser cáncer cervicouterino.
CASO CLÍNICO 3
7. La etapa clínica de acuerdo a la FIGO es:
IB Hombre de 25 años que acude con usted por tener aumento de
IIA tamaño del testículo derecho de 3 meses de evolución, no
IIB doloroso. Niega otra sintomatología. A la exploración física usted
IIIB detecta testículo derecho aumentado de tamaño, de consistencia
dura, no doloroso, transiluminación negativa. Sin otros hallazgos
De acuerdo a la clasificación de la FIGO (2009), la presencia de relevantes a la exploración física.
hidronefrosis clasifica a un paciente en etapa IIIB.
11. Su diagnóstico presuntivo es:
8. El factor de riesgo más importante es: Hernia inguinoescrotal
Tabaquismo Cáncer de testículo
Infección por VPH Varicocele
Múltiples parejas sexuales Orquiepididimitis
Multiparidad
Se trata de un hombre con edad de riesgo, con aumento de
Evidencia señala que esta neoplasia se debe hasta en 98% a la volumen sin dolor, duro y transiluminación negativa, todos estos
infección por VPH, por lo que se considera como el factor hallazgos clínicos son compatibles con cáncer testicular.
etiológico. La edad, el tabaquismo, la multiparidad y las múltiples
parejas sexuales se consideran otros factores de riesgo importante. 12. Para confirmar su diagnóstico usted solicita:
Marcadores tumorales y ultrasonido testicular
9. Los serotipos de VPH que incluye la vacuna cuadrivalente son: Radiografía de abdomen
6, 11, 16, 18 Biometría hemática y urocultivo
16, 18, 31, 33 Es un diagnóstico clínico
6, 11, 31, 33
16, 17, 18, 20 Ante la sospecha de cáncer testicular, los estudios iniciales son el
uso de ultrasonido testicular y los marcadores tumorales.
La vacuna cuadrivalente (Gardasil), protege contra condilomas Posteriormente, la tomografía de tórax y abdomen se utilizan para
genitales, displasia y cáncer, al proteger contra los serotipos 6 y 11 realizar la estadificación. El diagnóstico definitivo se obtiene
(asociados a verrugas vaginales) y 16 y 18 (los más comunes que mediante el análisis histopatológico de la orquiectomía.
causan enfermedad maligna).
13. Los marcadores tumorales útiles en cáncer de testículo son:
10. El tratamiento de esta paciente debe ser: Ca19.9, fracción B de la GCH y DHL
Alfafetoproteína, fracción B de la GCH y DHL física destaca ictericia de piel y mucosas, no tiene megalias, pero
Alfafetoproteína, DHL y Ca125 usted detecta a la exploración física signo de Courvoisier, dolor a
Ca 19.9, Ca 15.3 y DHL la palpación en epigastrio, sin palpar alguna masa.

Los marcadores tumorales en caso de sospecha de tumor testicular 16. Su diagnóstico presuntivo es:
son alfafetoproteína, fracción B de la gonadotropina y DHL. Cáncer de vesícula biliar
Cáncer de páncreas
14. Los estudios de extensión confirman enfermedad confinada Hepatitis aguda
al testículo. El tratamiento debe ser: Coledocolitiasis
Orquiectomía radical a través del escroto
Orquiectomía radical a través de la ingle Se trata de un paciente con factores de riesgo (tabaquismo),
Quimioterapia además de pérdida de peso e ictericia, lo que debe hacer
Antibióticos y reevaluación del paciente sospechar un tumor maligno. La presencia del signo de Courvoisier
debe ser un auxiliar para sospechar la presencia de un tumor
El tratamiento quirúrgico de elección de los pacientes con cáncer pancreático.
testicular es la orquiectomía radical inguinal. El uso de
quimioterapia está reservado a pacientes con etapas clínicas 17. El signo de Courvoisier es:
avanzadas o con riesgo de recurrencia. Hepatomegalia dolorosa
Vesícula biliar no dolorosa palpable
15. Posterior a la cirugía continúa con marcadores tumorales Ganglio periumbilical palpable
positivos, se realizan nuevos estudios que muestran metástasis Ganglio supraclavicular palpable
pulmonares y ganglios retroperitoneales. El tratamiento a seguir
es: El signo de Courvoisier consiste en la palpación de la vesícula biliar
Quimioterapia de forma indolora y se encuentra presente hasta en el 25% de los
Radioterapia pacientes con cáncer de páncreas.
Resección de metástasis pulmonares
Tratamiento paliativo 18. El factor de riesgo más importante para la patología que
usted considera en su diagnóstico es:
En el caso de un paciente que tiene enfermedad etapa III, con Edad
metástasis pulmonares, el tratamiento de elección de este Tabaquismo
paciente es quimioterapia. El esquema y número de quimioterapias Infección por VHC
dependerá del grupo pronóstico en el que se encuentre este Historia familiar
paciente y si es un tumor germinal seminomatoso o no Hasta 30% de los casos de cáncer de páncreas se asocian a consumo
seminomatoso, pero de forma general debe recibir quimioterapia de tabaco y se considera el principal factor de riesgo. La edad es
con intento curativo y con altas posibilidades de éxito. otro factor de riesgo importante, así como el consumo de alcohol y
la historia familiar.
CASO CLÍNICO 4
19. Dentro de las mutaciones genéticas en cáncer de páncreas,
Hombre de 60 años que acude a su consulta por presentar dolor la más frecuente es:
abdominal de 4 meses de evolución, acompañado de pérdida de BRCA
peso e ictericia de 3 semanas de evolución. Cuenta con K-Ras
antecedente de tabaquismo intenso por 20 años. A la exploración Myc
MSH1 Adenosarcoma

En el caso de cáncer de páncreas, entre 75-90% de los pacientes Más del 90% de las neoplasias malignas del colon corresponden a
presentan en el tumor mutación de k-Ras. También la mutación de adenocarcinomas.
BRCA, MSH1 y p53 se han asociado a cáncer de páncreas, aunque
en menor proporción, ya que entre todas representan no más del 23. El marcador tumoral que puede elevarse en estos casos es:
10% de los casos de cáncer de páncreas. Alfafetoproteína
Ca 19.9
20. La localización más frecuente del cáncer de páncreas es: Antígeno carcinoembrionario (ACE)
Cabeza Ca 15.3
Cuerpo
Proceso uncinado El marcador tumoral que puede elevarse y es útil en el seguimiento
No existe una preferencia de los pacientes es el ACE, con una sensibilidad del 80% y una
especificidad del 90% durante el seguimiento.
65-80% de los tumores pancreáticos se localizan en la cabeza del
páncreas, 20% en el cuerpo o cola y 5-15% son difusos. 24. Los siguientes son factores de riesgo para cáncer de colon
excepto:
CASO CLÍNICO 5 AINEs
Enfermedad inflamatoria intestinal
Hombre de 65 años de edad acude a tu consultorio por presentar Síndrome de Lynch
dolor abdominal predominantemente en fosa iliaca derecha, Consumo elevado de carnes rojas
además de hematoquezia de 3 meses de evolución, acompañado de
pérdida de peso de 7kg en 3 meses. A la exploración física con Se ha observado en varios estudios epidemiológicos que el consumo
palidez de tegumentos, se palpa tumoración en fosa iliaca de AINEs, principalmente aspirina, disminuye el riesgo de
derecha. Se realiza colonoscopia con lesión exofítica en colon desarrollar cáncer colorrectal. El resto se considera factores de
ascendente de 5cm. riesgo.

21. Su diagnóstico presuntivo es: 25. El escrutinio de cáncer de colon debe empezar a los:
Enfermedad inflamatoria intestinal 40 años
Cáncer de colon 45 años
Carcinoide apendicular 50 años
Tuberculosis colónica 55 años

Este paciente tiene un cuadro clínico de dolor abdominal, De acuerdo a las guías, el escrutinio de cáncer de colon debe
hematoquezia y pérdida de peso, es mayor de 50 años y se iniciar tanto en hombres como mujeres a partir de los 50 años.
encuentra una lesión exofítica en el colon, por lo que la sospecha
diagnóstica debe ser malignidad. CLINICAL CASE 6

22. El tipo histológico más frecuente de cáncer de colon es: A 49-year-old man noticed a lesion on his right upper back. Over
Células en anillo de sello time, the lesion has grown, turned darker and raised in
Adenocarcinoma appearance. The patient was referred to you. Physical
Leiomiosarcoma examination showed a 1.5cm skin lesion, with irregular borders
and variegated color, asymmetrical, with no other abnormalities. melanoma, siendo peor el pronóstico a mayor número de ganglios
No lymphadenopathy. afectados.

26. Which one is your most likely diagnosis? 30. Patient’s treatment must include:
Malignant melanoma Wide excision and sentinel lymph node biopsy
Basocelular carcinoma Wide excision and lymphadenectomy
Actinic keratosis Immunotherapy
Dermatophytosis Chemotherapy

Se trata de una lesión que ha mostrado crecimiento, además de El tratamiento de elección en los pacientes que no se encuentran
pigmentación irregular, asimetría y bordes irregulares, por lo que la con linfadenopatías al diagnóstico, es la resección del tumor con
sospecha clínica debe ser melanoma maligno. márgenes negativos con biopsia de ganglio centinela. Si el ganglio
centinela es negativo, no se ha observado beneficio en realizar
27. One of these, isn’t a risk factor for melanoma: linfadenectomía en este grupo de pacientes.
Age
Ultraviolet B rays exposure CLINICAL CASE 7
Smoking
Familial melanoma A 40-year-old healthy female has noticed a "lump" in her neck, so
she is seeking an evaluation. She has no symptoms. Physical
El tabaquismo, a pesar de que puede causar cambios cutáneos y es examination reveals an asymmetric thyroid with a nodule
factor de riesgo para otras neoplasias, no se ha encontrado como (approximately 1.5cm in size) on the right side of the neck, no
un factor independiente para el desarrollo de melanoma. lymphadenopathy. She also has thyroid function tests, with normal
results.
28. Which is the most frequent subtype of melanoma in Mexico?
Nodular melanoma 31. What would be your next step?
Lentigo malign melanoma Perform anti-thyroid antibodies
Acral lentiginous melanoma Start non-steroidal anti-inflammatory drugs and reassurance
Amelanic melanoma Thyroid ultrasound and needle aspiration biopsy
Start levothyroxine 1.6 mcg per kilogram
En México, a diferencia de otras regiones del mundo, el melanoma
acral lentiginoso es el tipo más común, con más del 50% de los Ante la presencia de un nódulo tiroideo en un paciente eutiroideo,
casos. lo que se recomienda es realizar un ultrasonido con una biopsia por
aspiración con aguja fina.
29. The most important prognostic factor in melanoma is:
Clark level 32. Which is the most frequent histologic type of thyroid
Breslow level cancer?
Lymph nodes with metastases Papillary
Ulceration Follicular
Medullary
La presencia de metástasis en los ganglios linfáticos regionales es Hürtle cell
el factor pronóstico más importante en los pacientes con
El carcinoma papilar representa del 75-85% de los tumores Gastric cancer
tiroideos, seguido por el carcinoma folicular (10-15%). Gastro esophageal reflux disease
Peptic ulcer disease
33. The most important prognostic factor in thyroid cancer is: Pancreatitis
Lymph nodes with metastases
Age Se trata de un paciente con dispepsia, pérdida de peso, dolor
Negative surgical borders abdominal y melena, por lo que la sospecha diagnóstica inicial
Histologic grade debe ser malignidad. Por la presencia de la sintomatología la
sospecha debe ser más gástrico que esofágico.
El factor pronóstico más importante es la edad, lo pacientes
después de los 40-45 años presentan mayor agresividad local y 37. What would you do to confirm your presumptive diagnosis?
mayor mortalidad. Esophagogastroduodenoscopy and biopsy
Abdominal CT
34. Biopsy reports: medullary thyroid cancer. What genes could Double-contrast barium swallow
have a mutation? pHmetry and manometry
HER2
RET Ante la sospecha de un tumor gástrico, la endoscopia debe ser el
Myc estudio inicial, ya que posibilita la inspección de la mucosa gástrica
Ras y permite la toma de biopsia para confirmar el diagnóstico.

Las mutaciones en el gen RET se asocian a cáncer medular de 38. The most common histological subtype of gastric cancer is:
tiroides, ya sean mutaciones de novo, o asociadas a síndromes Lymphoma
genéticos como MEN2. Adenocarcinoma- intestinal type
35. Which one of the following syndromes has increased risk of Adenocarcinoma-diffuse type
medullary thyroid cancer? Leiomyosarcoma
Multiple endocrine neoplasia type 1
Multiple endocrine neoplasia type 2 Los adenocarcinomas representan el 95% de las neoplasias
Hashimoto’s thyroiditis gástricas, siendo la variante más frecuente la de tipo intestinal.
Familial papillary thyroid cancer
39. The most frequent localization of gastric cancer is:
Entre 20-25% de los tumores medulares de tiroides se asocian a Cardia
síndromes hereditarios, principalmente la neoplasia endócrina Greater curvature
múltiple tipo 2. Pyloric area
Body and fundus
CLINICAL CASE 8
La localización más frecuente es el área pilórica, el 50% de los
A 70-year-old woman comes to your practice with a complaint of tumores se localizan en esa región, seguida por la curvatura menor
dyspepsia and weight loss. She also refers early satiety, abdominal (25%), cuerpo y fondo (10%) y cardias (10%).
pain with nausea, and melena. Physical examination: skin pallor, no
other physical findings. 40. Hereditary gastric cancer has been associated with
mutations of:
36. Which one is your most likely diagnosis? HER2
Ras Endometrioid
E-cadherin Mucinous
BRCA 1
El tipo endometrioide es el más frecuente de cáncer endometrial,
El síndrome de cáncer gástrico hereditario se asocia a mutaciones representa entre 60-65%.
en el gen de E-cadherina.
44. The initial treatment for this patient is:
CLINICAL CASE 9 Chemotherapy plus radiotherapy
Simple hysterectomy
A 70-year-old women with diabetes mellitus and hypertension Uterine curettage
presents to your practice with abnormal uterine bleeding since 3 Total abdominal hysterectomy and bilateral salpingo-oophorectomy
months ago. Physical examination with BMI 35kg/m2, no other with peritoneal biopsy and cytology
physical findings.
El tratamiento quirúrgico del cáncer de endometrio debe incluir
41. Which is your most likely diagnosis? histerectomía total abdominal con salpingooforectomía bilateral,
Ovarian cancer biopsia de epiplón mayor y exploración de la cavidad abdominal.
Cervical cancer
Endometrial cancer 45. One of the following syndromes has been associated with
Vaginal cancer endometrial cancer:
Lynch syndrome
Se trata de una mujer postmenopáusica, obesa y diabética con Multiple endocrine neoplasia type 1
sangrado uterino anormal, por lo que se debe tener una alta Li-Fraumeni syndrome
sospecha de malignidad, en este caso, por la sintomatología, la BRCA mutations
primera sospecha debe ser cáncer de endometrio.
El síndrome de Lynch se encuentra como causa entre el 2-8% de los
42. One of these, is not a risk factor for endometrial cancer: tumores endometriales, presentándose en estas pacientes a menor
Smoking edad.
Obesity
Polycystic ovary disease CLINICAL CASE 10
Tamoxifen
A 17-year-old male presented with increasing pain in the left
El tabaquismo se ha asociado con menor riesgo de desarrollar upper arm of approximately 3 months' duration and a recent onset
cáncer de endometrio en mujeres postmenopáusicas, obviamente, of low-grade fever. On physical examination, there was some local
el riesgo de salud asociado con el tabaquismo sobrepasa este tenderness and soft tissue swelling over the proximal and mid
beneficio. Sin embargo, se ha asociado como reductor a través de thirds of the left humerus. Plain radiograph shows a large ill-
estimular el metabolismo hepático de los estrógenos. Los demás defined, destructive, diaphyseal intramedullary lesion with
son factores de riesgo asociados a cáncer de endometrio. permeate pattern of bone destruction and periosteal reaction of
an “onion-peel” type. The lesion is associated with a soft tissue
43. Which is the most frequent histological type of endometrial mass. Biopsy material showed a highly cellular, infiltrative
cancer? neoplasm consisting of sheets of tightly packed, round cells with
Squamous cell very scant cytoplasm ("round blue cell tumor").
Papillary serous
46. Which one is your most likely diagnosis? Only surgery
Osteosarcoma Only chemotherapy
Ewing sarcoma Chemotherapy and radiotherapy
Leiomyosarcoma Chemotherapy, radiotherapy and surgery
Fracture of left humerus
El tratamiento de los sarcomas de Ewing debe ser multidisciplinario
El sarcoma de Ewing se presenta con síntomas constitucionales e incluye quimioterapia, generalmente de forma neoadyuvante,
como fiebre de bajo grado, además de apariencia de capa de cirugía y radioterapia.
cebolla en la radiografía. Dentro de la patología se observa un
tumor con células redondas, pequeñas y azules.
Cáncer en la mujer
47. Which one of the following translocations has been
associated with your diagnosis? CASO CLÍNICO 1
9; 22
11; 22 Mujer de 45 años de edad, previamente sana. Antecedentes
8; 14
14; 18 ginecoobstétricos: Menarca a los 13 años, inicio de vida
sexual: 15 años, Parejas sexuales: 6; Gestas 3, partos 3.
Del 85-90% de los tumores de Ewing presentan la translocación t Niega consumo de anticonceptivos orales. Citología vaginal
(11; 22) (q24; q12) que forma una proteína de fusión anormal nunca realizada. Refiere que inició desde hace 4 meses con
denominada EWS-FLI. sangrado transvaginal anormal, inicialmente posterior a
tener relaciones sexuales y actualmente con sangrado
48. The most frequent site of metastatic disease in bone transvaginal cada 15 días. Exploración cardiopulmonar y de
sarcomas is: abdomen sin alteraciones. Tacto vaginal: Se palpa lesión
Liver friable en el cérvix, de aproximadamente 6cm, con sangrado
Lymph nodes en el guante explorador. Resto normal.
Lung
Brain
1. ¿Cuál es el tipo histológico más frecuente del cáncer
Hasta el 80% de las metástasis de los sarcomas óseos son al pulmón. cervicouterino?
Células claras
49. One of these is not a risk factor for bone sarcomas: Adenocarcinoma
Li-Fraumeni syndrome Carcinoma epidermoide
Familial cancer Carcinosarcoma
Age
Mutant p53 El tipo histológico más frecuente es el carcinoma
epidermoide o de células escamosas (80-90%), seguido del
Aunque la mayor parte de los sarcomas óseos se presentan en adenocarcinoma y el carcinoma adenoescamoso.
pacientes jóvenes, no se ha asociado a la edad como factor de
riesgo para la presencia de estos tumores.
2. ¿Cuáles son los tipos de VPH a los que se asocia el
50. Treatment of this patient must include: cáncer cervicouterino?
6, 11, 16, 18 El tratamiento de las etapas localmente avanzadas (IIB, III,
16, 18, 31, 33 IVA) es quimiorradioterapia definitiva, principalmente con
11, 18, 41, 45 esquemas basados en platino.
6, 11, 42, 43
CASO CLÍNICO 2
En las mujeres mexicanas se han identificado a los tipos de
alto riesgo 16 y 18 como los más frecuentes, presentes hasta Mujer de 32 años, sin antecedentes relevantes. Acude a
en el 80% de los casos, los siguientes en frecuencia son 31, centro de salud a realizarse citología cervicovaginal de
33, 35, 39, 45, 51, 52, 56 y 58. escrutinio. Refiere haber tenido 2 parejas sexuales, no ha
tenido embarazos previos.
3. ¿Qué estudios de extensión se recomienda realizar?
Cistoscopia, colonoscopia y colposcopia 6. Los siguientes son factores de riesgo para cáncer
Rectosigmoidoscopia, urografía excretora, cistoscopia, cervicouterino, excepto:
radiografía de tórax Antecedente de enfermedad de transmisión sexual
Sólo requiere estadiaje clínico Inicio de vida sexual después de los 18 años
Colposcopia y tomografía del tórax Múltiples parejas sexuales
Nunca haberse realizado una citología cervicovaginal
Los estudios de extensión que se deben solicitar de acuerdo a
la FIGO, son la rectosigmoidoscopia, urografía excretora, Los factores de riesgo son bajo nivel sociocultural, inicio de
radiografía de tórax y cistoscopia si se sospecha afección vida sexual temprana, múltiples parejas sexuales,
vesical o rectal. tabaquismo y el hecho de nunca haberse realizado una
citología.
4. En los estudios se documenta hidronefrosis bilateral. ¿A
qué etapa clínica corresponde? 7. De acuerdo a la Norma Oficial Mexicana, se recomienda
IA realizar citología cervicovaginal:
IIIA De los 20 a los 64 años, cada 2 años
IIIB De los 25 a los 64 años, anual
IV De los 25 a los 64 años, cada 2 años
De los 20 a los 60 años, anual
De acuerdo a la clasificación de la FIGO del 2009, la
presencia de hidronefrosis clasifica a la paciente en etapa Si bien existen distintas guías de práctica, tanto nacionales
IIIB. como internacionales, lo que establece la Norma Oficial
Mexicana NOM-014-SSA2-1994, modificada en 2007, es que la
5. ¿Cuál es el tratamiento de esta paciente? citología debe solicitarse a las mujeres entre los 25 a los 64
Quimiorradioterapia definitiva años, en especial con factores de riesgo, de forma anual.
Cirugía + quimioterapia adyuvante
Cirugía + radioterapia adyuvante 8. En esta paciente se reporta citología cervicovaginal con
Nefrostomías bilaterales y cuidados paliativos lesión intraepitelial escamosa de alto grado. La incidencia
de NIC3 es aproximadamente:
20% menstruación a los 51 años. Nulípara. Acude por presentar
30% una lesión en mama derecha, pétrea, de 3 cm de diámetro
40% desde hace 3 meses. A la exploración física detecta usted
50% además la presencia de un conglomerado ganglionar fijo,
pétreo en la axila ipsilateral. Se realiza biopsia con reporte
La incidencia de NIC 3 en pacientes con lesión intraepitelial histopatológico de cáncer de mama, receptores hormonales
de alto grado por citología cervicovaginal es positivos, HER2 positivo.
aproximadamente del 50%.
11. El tipo histológico más frecuente de cáncer de mama
9. El manejo terapéutico de la NIC3 en esta paciente es: es:
Vigilancia con citología cervicovaginal en 3 meses Ductal
Vigilancia con citología cervicovaginal en 6 meses Mixto
Crioterapia o conización Medular
Histerectomía simple Sarcoma

Una vez confirmado el diagnóstico histológico de NIC3, los El tipo histológico más frecuente del cáncer de mama es el
métodos ablativos o escisionales son las terapéuticas ductal, que representa hasta el 80% de los casos, seguido por
aceptadas, entre ellas crioterapia, electrocoagulación, el lobulillar (10-15%). Las otras variedades histológicas son
conización; dado la alta tasa de persistencia o progresión, si raras.
está descartada la invasión y tiene colposcopia satisfactoria.
En mujeres con paridad satisfecha histerectomía total. 12. Los estudios de extensión para descartar enfermedad
metastásica que se recomiendan en este caso son:
10. La vacuna cuádruple contra VPH protege contra los Ninguno porque se trata de una etapa temprana
siguientes serotipos: Gammagrama óseo, radiografía de tórax y ultrasonido
6, 11, 16, 18 hepático
6, 16, 26, 31 Mastografía y ultrasonido mamario
11, 18, 41, 45 Tomografía de tórax y abdomen
6, 11, 42, 43 En este caso, al haber presencia de un conglomerado
ganglionar en la axila, se trata de una etapa localmente
La vacuna cuadrivalente (Gardasil) protege contra los avanzada y está indicada la realización de estudios de
serotipos 6, 11, 16 y 18, protegiendo contra los condilomas y extensión. Los estudios indicados de acuerdo a la AJCC son
la displasia/cáncer, mientras que la vacuna bivalente gammagrama óseo, radiografía de tórax y ultrasonido
(Cervarix) protege contra lo serotipos 16 y 18 (aunque se ha hepático.
observado protección cruzada contra los serotipos 31 y 35).
13. Los estudios de extensión no revelen evidencia de
CASO CLÍNICO 3 metástasis. De acuerdo al TNM, esta paciente se encuentra
en una etapa:
Mujer de 60 años, previamente sana. Antecedentes T1N1M0
ginecoobstétricos: Menarca a los 12 años, fecha de última T2N1M0
T2N2M0 recientemente secreción sanguinolenta por el pezón. A la
T3N2M0 exploración física con nódulo mamario de 1.5cm en
cuadrante superior externo, no se palpan ganglios axilares.
De acuerdo al TNM, la paciente se encuentra en una etapa T2 Se realiza el diagnóstico de cáncer de mama.
(tumor mayor a 2cm y hasta 5cm), N2 (conglomerado axilar
ganglionar fijo), M0 (se descartaron metástasis en los 16. ¿Cuál se descarta como un factor de riesgo para cáncer
estudios de extensión), lo cual corresponde a una etapa de mama?
clínica IIIA. Historia familiar de cáncer de mama
Menopausia tardía
14. El factor pronóstico más importante en esta paciente Lactancia materna
es: Hiperplasia ductal atípica
Edad
Ganglios positivos Dentro de los factores de riesgo de cáncer de mama, se
Tamaño del tumor encuentran la historia familiar de cáncer de mama, la
HER2 positivo menarca temprana, la menopausia tardía y la nuliparidad.
Lesiones premalignas como la hiperplasia ductal atípica y el
La presencia de ganglios linfáticos con infiltración maligna es carcinoma ductal in situ se consideran también factores de
el factor pronóstico más importante en cáncer de mama, riesgo. La lactancia materna se considera un factor protector
siendo la supervivencia global a 5 años 85% en las pacientes para el desarrollo de cáncer de mama.
con ganglios axilares negativos versus aproximadamente 50%
en las pacientes con ganglios positivos (+ número de ganglios 17. De acuerdo a la Norma Oficial Mexicana, se
afectados, -- supervivencia). recomienda realizar mastografía:
De los 40 a los 69 años, anual
15. El medicamento que debe incluir el tratamiento de De los 40 a los 69 años, cada 2 años
esta paciente por tener expresión de Her2 es: De los 50 a los 69 años, anual
Doxorrubicina De los 50 a los 69 años, cada 2 años
Trastuzumab
Bevacizumab La NOM-041-SSA2-2011 recomienda la mastografía de los 40 a
Rituximab los 69 años, cada 2 años.

El trastuzumab es un medicamento antiHER2 que ha probado 18. Dentro de la mortalidad por cáncer, el cáncer de mama
mejorar la supervivencia libre de progresión y la se encuentra en el lugar:
supervivencia global en las pacientes con sobre expresión de 1
HER2 en cáncer de mama. 2
CASO CLÍNICO 4 3
4
Mujer de 45 años, sin antecedentes relevantes. Acude por
presentar nódulo mamario de 5 meses de evolución que ha
aumentado de tamaño de forma progresiva, se agregó
A partir del 2006, el cáncer de mama superó al cáncer citología vaginal hace 2 años negativa a malignidad. Refiere
cervicouterino como la primera causa de muerte por cáncer haber iniciado con sangrado transvaginal hace 3 meses, sin
en mujeres mexicanas. otra sintomatología. Exploración física: IMC 35, sin otras
19. Los estudios de extensión para descartar enfermedad alteraciones en la exploración física.
metastásica que se recomiendan en este caso son:
Resonancia magnética de la mama 21. El estudio inicial de esta paciente para confirmar su
Mastografía y ultrasonido mamario sospecha diagnóstica es:
Tomografía de tórax y abdomen Tomografía de abdomen y pelvis
Ninguno porque se trata de una etapa temprana Ultrasonido pélvico y toma de biopsia de cavidad
endometrial
En el caso de esta paciente se trata de una etapa temprana Resonancia magnética de pelvis
cuya posibilidad de enfermedad metastásica es baja, por lo Prueba terapéutica con progestágenos
que los estudios de extensión no se recomiendan de rutina, a
menos que presente síntomas o se encuentren anormalidades En este caso, dado que es una mujer postmenopáusica,
de estudios de laboratorio como elevación de las PFH o de la obesa, hipertensa y diabética con sangrado uterino anormal,
fosfatasa alcalina. la sospecha debe ser patología endometrial, principalmente
cáncer de endometrio. Ante la sospecha, lo indicado es
20. El tratamiento inicial de esta paciente debe ser: realizar un ultrasonido para evaluar el grosor del endometrio
Cirugía conservadora y ganglio centinela y una biopsia de la cavidad endometrial para confirmar el
Cirugía conservadora y disección axilar diagnóstico.
Mastectomía y ganglio centinela
Mastectomía y disección axilar 22. El tipo histológico más frecuente de cáncer
endometrial es:
Si no existe contraindicación para recibir radioterapia, Seroso papilar
enfermedad multicéntrica y dificultad para obtener Endometrioide
márgenes adecuados o un resultado cosmético aceptable, se Células claras
puede realizar cirugía conservadora. Dado que la paciente Adenoescamoso
tiene ganglios clínicamente negativos, se puede realizar la
exploración del ganglio centinela. En este caso la paciente El tipo endometrioide es el más frecuente (60-65%). El
aparentemente tiene sólo un tumor, por lo que se podría carcinoma mucinoso representa menos del 1% de los casos,
realizar cirugía conservadora. mientras que el de células claras representa hasta el 4% y el
seroso papilar del 1-10% de los casos.
CASO CLÍNICO 5
23. Dentro de los factores de riesgo conocidos para cáncer
Mujer de 65 años, cuenta con antecedente de diabetes de endometrio se encuentran los siguientes, excepto:
mellitus, hipertensión arterial sistémica en tratamiento. Diabetes mellitus
Antecedentes ginecoobstétricos: Menarca a los 15 años, Uso de Tamoxifen
Gestas 3, Partos 3, Fecha última menstruación a los 50 años, Edad menor de 50 años
con terapia de reemplazo hormonal por 5 años, última Consumo de estrógenos en menopausia
Los factores de riesgo se relacionan con la exposición a peritoneal, además de lesión ovárica izquierda, sólida,
estrógenos, como la terapia de reemplazo hormonal, la hipervascular, de 6cm de diámetro.
obesidad, nuliparidad. Otros factores de riesgo son la
obesidad, la diabetes y la hipertensión. El 75% de casos se 26. El tipo histológico más frecuente de cáncer de ovario
diagnostican en mujeres mayores de 50 años. es:
24. Una de las siguientes afirmaciones sobre el cáncer de Tumor germinal
endometrio no es cierta: Carcinoma seroso papilar
La mayor parte de las pacientes se presenta con sangrado Cistadenocarcinoma mucinoso
transvaginal anormal Carcinoma de células claras
La mayor parte de las pacientes se diagnostica en etapas
avanzadas de metástasis Las neoplasias ováricas más frecuentes son las epiteliales,
La exploración física en general no muestra datos relevantes siendo los cistadenocarcinomas serosos los más frecuentes
Dentro de los factores pronósticos se encuentran el grado, la (30%), seguidos el endometrioide (10-20%) y el mucinoso
invasión al miometrio y la etapa clínica (10%). El tumor de células claras representa del 5-10%, el
epidermoide, menos del 1%.
La mayor parte de las pacientes se presentan con 27. El marcador tumoral que puede usted solicitar para
enfermedad confinada al útero. El resto de las aseveraciones complementar su sospecha diagnóstica es:
son correctas. Antígeno carcinoembrionario
Alfafetoproteína
25. Se considera la piedra angular del tratamiento en el Ca 125
cáncer de endometrio: Ca 15.3
Quimioterapia
Quimioterapia + radioterapia En las neoplasias epiteliales ováricas, uno de los marcadores
Cirugía tumorales de mayor utilidad es el Ca 125, que se encuentra
Braquiterapia elevado hasta en el 85% de los tumores ováricos. Sólo en el
caso de mujeres <40 años, que se sospeche la presencia de
La cirugía es fundamental en el tratamiento ya que evalúa la un tumor germinal de ovario, se debe solicitar
extensión de la enfermedad, permite la estadificación y es alfafetoproteína, fracción β de la gonadotropina y DHL.
terapéutica.
28. La siguiente aseveración es cierta con respecto a los
CASO CLÍNICO 6 marcadores tumorales en cáncer de ovario:
La elevación del Ca125 es diagnóstica de cáncer de ovario,
Mujer de 65 años, previamente sana. Acude por presentar ya que se eleva hasta en el 80% de los casos
desde hace 5 meses pérdida de peso y aumento del La elevación del Ca19.9 es diagnóstica de cáncer de ovario,
perímetro abdominal. A la exploración física se documenta ya que se eleva hasta en el 80% de los casos
ascitis, no a tensión. Se realiza ultrasonido abdominal, con El Ca125 puede elevarse por causas no malignas que causen
hígado de bordes regulares, sin alteraciones sonográficas, irritación peritoneal, en el embarazo y menstruación
ascitis y múltiples imágenes compatibles con carcinomatosis El Ca125 no es útil para evaluar respuesta a tratamiento ni
para detección de recurrencia
Aunque el Ca125 se eleva hasta en el 85% de los casos, este Cáncer gastrointestinal
marcador no es específico de cáncer de ovario y se
encuentra elevado en todas aquellas condiciones no CASO CLÍNICO 1
oncológicas que se presentan con inflamación del peritoneo Hombre de 65 años, cuenta con antecedente de tabaquismo
como pancreatitis, diverticulitis, embarazo, menstruación, y alcoholismo intenso. Refiere que desde hace 4 meses inició
así como otras neoplasias como cáncer de mama, páncreas, con disfagia, pérdida de peso y dolor retroesternal. Se
hígado y pulmón. Otra utilidad del Ca125 es evaluar la realizó endoscopia con hallazgo de lesión exofítica sugestiva
respuesta a tratamiento, además, es de utilidad en la de cáncer de esófago.
detección temprana de recurrencia. 1. El tipo histológico más frecuente de cáncer de
esófago es:
29. Estos son factores de riesgo para cáncer de ovario, Adenocarcinoma
excepto: Carcinoma epidermoide
Edad mayor de 50 años Adenoescamoso
Factores genéticos Carcinoma con células en anillo de sello
Menarca temprana y menopausia tardía
Anticonceptivos orales por más de 5 años El tipo histológico más frecuente de cáncer de esófago
La duración de anticonceptivos orales por más de 5 años se es el carcinoma epidermoide (50%), seguido del
considera un factor protector, ya que se ha encontrado en adenocarcinoma (42%).
estudios que tiene un riesgo relativo de 0.58. El resto se
considera factores de riesgo para cáncer de ovario.
2. Los factores de riesgo para presentar cáncer de
esófago son los siguientes, excepto:
30. El tratamiento inicial del cáncer de ovario debe ser:
Tabaquismo
Radioterapia abdominal
Alcoholismo
Cirugía con resección de lesión ovárica
Reflujo gastroesofágico
Cirugía radical con histerectomía, salpingooforectomía
Infección por Helicobacter pylori
bilateral, omentectomía y biopsias de correderas
parietocólicas
Los principales factores de riesgo para presentar cáncer de
Quimioterapia y radioterapia
esófago son tabaquismo, alcoholismo, ingesta de
En los tumores epiteliales de ovario, la cirugía es muy nitrosaminas, acalasia o estenosis esofágica, reflujo
importante para disminuir la carga tumoral y determinar el gastroesofágico y esófago de Barrett. La infección por
estado clínico. La cirugía ideal es una cirugía conocida como Helicobacter pylori no representa un factor de riesgo.
rutina de ovario, donde se realiza lavado en los cuatro
cuadrantes y se envía a citología, además de exploración de
3. La localización anatómica más frecuente del
la cavidad peritoneal, ganglios pélvicos y paraaórticos,
carcinoma epidermoide es:
regiones subdiafragmáticas, histerectomía con
Tercio superior
salpingooforectomía bilateral, omentectomía, biopsias de
Tercio medio
correderas parietocólicas, fondo de saco, peritoneo vesical,
Tercio inferior
ambos hemidiafragmas.
Unión gastroesofágica supraclavicular izquierda, cardiopulmonar sin alteraciones,
abdomen con dolor a la palpación en epigastrio, además de
La localización más frecuente del carcinoma epidermoide es aumento del perímetro abdominal a expensas de ascitis.
en el tercio medio (50-65% de los casos), seguido por el 6. El tipo histológico más frecuente de cáncer gástrico
tercio inferior (25-35%). La localización más común del es:
adenocarcinoma es el tercio inferior 60%. Adenocarcinoma tipo intestinal
Adenocarcinoma tipo difuso
Carcinoma epidermoide
4. El síntoma más frecuente del cáncer de esófago es: Tumor del estroma gastrointestinal
Pérdida de peso
Disfagia El adenocarcinoma el tipo histológico más común de cáncer
Odinofagia gástrico y dentro de estos el adenocarcinoma intestinal es el
Disfonía más frecuente, desarollado a partir de lesiones
precancerosas. Este tipo de cáncer gástrico se encuentra más
El cuadro clínico de un paciente con carcinoma esofágico es frecuentemente en hombres de edad avanzada.
presencia de disfagia, odinofagia, disfonía y pérdida de peso
principalmente. El síntoma más común es la presencia de 7. Los siguientes son factores de riesgo para cáncer
disfagia, encontrado en alrededor de 75% de los casos. gástrico excepto:
Consumo de alimentos ahumados
Consumo alto de nitratos
Infección por Helicobacter pylori
5. El sitio más frecuente de metástasis es: Dieta rica en vitamina A y C
Hígado
Pulmón Los principales factores de riesgo para padecer cáncer
Hueso gástrico son: consumo de alimentos con nitritos, alimentos
Pleura ahumados, infección por Helicobacter pylori, tabaquismo,
bajo consumo de proteínas y grasas y mutaciones en E-
Las metástasis a hígado son las que más frecuentemente se
encuentran en pacientes con cáncer esofágico. También es caderina. La dieta rica en vitamina A y C no son factores de
frecuente encontrar metástasis a pulmón, peritoneo, hueso, riesgo.
cerebro.
8. Las siguientes son características del carcinoma
gástrico intestinal, excepto:
CASO CLÍNICO 2 Es el más frecuente en la mujer
Mujer de 45 años de edad, sin antecedentes previos de Es más frecuente en población de mayor edad
importancia. Refiere haber iniciado desde hace 4 meses con Se asocia con factores como dieta y consumo de tabaco
evacuaciones melénicas, acompañadas de pérdida de peso de Es frecuente en regiones con cáncer gástrico epidémico
20 kg en dicho periodo de tiempo. A la exploración física con
palidez de tegumentos, además usted detecta adenopatía
El adenocarcinoma gástrico de variedad intestinal es el más endoscópico con una lesión ulcerada de bordes elevados
frecuentemente encontrado. Se presenta más localizada en la región prepilórica. Se toman biopsias con
frecuentemente en hombre y aumenta su incidencia a mayor reporte histopatológico de cáncer gástrico.
edad. 11. De acuerdo al reporte de la endoscopia,
corresponde a la clasificación de Bormann:
9. El cáncer gástrico ocupa el siguiente lugar dentro 1
de las causas de muerte por cáncer en México: 2
1 3
2 4
3
4 La endoscopia reporta una lesión ulcerada de bordes
El cáncer gástrico es la segunda causa de muerte por cáncer elevados, que corresponde al tipo 2 de la clasificación de
a nivel mundial, y la tercera en México. Aproximadamente Bormann.
50% de los pacientes presentan una etapa avanzada al
momento del diagnóstico. De las etapas tempranas 12. La localización más frecuente del cáncer gástrico
resecadas, 70% tienen metástasis a ganglios linfáticos, por lo es:
que la sobrevida a dos años se estima que es del 13.8%. Unión gastroesofágica
Curvatura mayor
10. La presencia de una adenopatía supraclavicular Curvatura menor
metastásica se denomina: Antro y píloro
Ganglio de la Hermana María José
Ganglio de Virchow En el antro y el píloro se localizan cerca de la mitad de los
Tumor de Krukenberg carcinomas gástricos.
Síndrome de Trousseau
13. Dentro de los estudios de extensión para descartar
La adenopatía supraclavicular es un signo de enfermedad enfermedad metastásica se recomienda:
avanzada. Se le denomina ganglio de Virchow. Los demás Resonancia magnética de abdomen y endoscopia
también son signos de enfermedad avanzada. El signo de la Tomografía tórax y abdomen, ultrasonido transendoscópico
hermana María José es un nódulo periumbilical; el tumor de Ultrasonido hepático, radiografía de tórax
Krukenberg es la metástasis a ovario; el síndrome de Biometría hemática y antígeno carcinoembrionario
Trousseau se refiere a la presencia de trombosis venosa
profunda. Dentro de los estudios complementarios se debe realizar TAS
de tórax y abdomen y ultrasonido endoscópico para valorar
profundidad de la invasión y estado ganglionar.
CASO CLÍNICO 3
Hombre de 56 años, que inició hace 6 meses con pérdida de 14. Se confirma que se trata de una lesión con invasión
peso y dolor epigástrico. Se inició manejo con inhibidor de hasta la muscular propia, sin adenopatías regionales
bomba de protones, sin mejoría clínica, por lo que se o enfermedad metastásica. La mejor conducta
solicitó un estudio endoscópico. Le reportan un estudio terapéutica es la siguiente:
Gastrectomía total El 85% de los casos se encuentran entre los 60-80 años, con
Gastrectomía parcial una mediana de 69 años, es más frecuente en el sexo
Quimioterapia masculino con una proporción 1.5:1. El 75% de los tumores se
Gastroyeyunoanastomosis tipo Billroth I ubican en la cabeza del páncreas. Al diagnóstico, sólo 15% de
los pacientes son candidatos a cirugía, 40% tiene enfermedad
La cirugía es el pilar terapéutico del tratamiento. Si la lesión irresecable y 45% tiene metástasis al diagnóstico. El 85% de
está localizada, sin adenopatías ni enfermedad metastásica, los tumores malignos corresponden a adenocarcinomas
el procedimiento indicado es la gastrectomía parcial. ductales.

15. La forma hereditaria de cáncer gástrico se asocia a 17. La localización anatómica más frecuente es:
mutaciones en: Cabeza
HER2 Proceso uncinado
E-cadherina Cuerpo
APC Cola
MLH1
El 75% de los tumores se ubica en la cabeza del páncreas,
La E-cadherina es una molécula de adhesión celular 11% se localizan en el cuerpo, 11% en la cola del páncreas y
transmembranal dependiente de calcio. Las mutaciones el resto son difusos.
de esta molécula se asocian con las formar hereditarias de
cáncer gástrico (carcinoma gástrico familiar) 18. Al momento de hacer diagnóstico de cáncer de
páncreas, lo más frecuente es que se encuentre
enfermedad:
CASO CLÍNICO 4 Localizada al páncreas
Hombre de 70 años, acude por presentar ictericia de 2 Con invasión vascular
semanas de evolución, acompañada de dolor abdominal en Localmente avanzada
epigastrio e hipocondrio derecho, además de pérdida de Metastásica
peso de 5kg en 2 semanas. Antecedente de tabaquismo y
consumo de alcohol intenso, diagnóstico de diabetes mellitus Al momento del diagnóstico, el 50% de los pacientes se
hace 3 meses. A la exploración física usted nota ictericia encuentra con metástasis. El 30% son localmente avanzados y
generalizada, cardiopulmonar sin alteraciones, abdomen con tan solo 8% de los pacientes presentan enfermedad resecable
vesícula palpable en hipocondrio derecho, no dolorosa. al diagnóstico.
16. El tipo histológico más frecuente de cáncer de
páncreas es: 19. Las siguientes aseveraciones son ciertas con
Adenocarcinoma acinar respecto al cáncer de páncreas, excepto:
Adenocarcinoma ductal El 30% se asocia a consumo de tabaco
Carcinoma epidermoide La mutación de k-ras es un suceso inicial en la carcinogénesis
Cistadenoma mucinoso Es más frecuente en mujeres
Existe predisposición familiar hasta en 8-10% de los casos
La mayoría de los casos se encuentran entre los 60-80 años. El Ca 19-9 se expresa en el 95% de la población. Su
Es más frecuente en el sexo masculino con una proporción sobreproducción se asocia a cáncer pancreático y del árbol
1.5:1. biliar, sin embargo, no es específico de estas entidades.
También se ha visto una sobreproducción en otras neoplasias
20.La palpación de la vesícula biliar no dolorosa se malignas (estómago, hígado, colon y recto) y en patologías
denomina como: benignas bilio-pancreáticas, como pancreatitis, colangitis y
Signo de Trousseau coledocolitiasis. Al no ser específica, no puede considerarse
Signo de Courvoisier como hallazgo suficiente para realizar el diagnóstico de
Triada de Charcot cáncer de páncreas.
Signo de Gray-Turner
23. El sitio más frecuente de metástasis es:
El signo de Courvosier- Terrier se refiere a la palpación de la Hueso
vesícula biliar en el hipocondrio derecho, sin inflamación de Hígado
su pared. Peritoneo
Pleura
21. El marcador tumoral útil dentro del abordaje
diagnóstico es: La metástasis a hígado es el sitio en donde se encuentran
Ca 125 hasta 70% de las metástasis en el cáncer de páncreas.
Ca 19.9
Antígeno carcinoembrionario 24. Se encuentra un tumor localizado en la cabeza del
Alfafetoproteína páncreas. ¿Cuál es el tratamiento inicial de este
paciente?
El Ca 19-9 (antígeno carbohidrato) se ha identificado Resección local de la lesión pancreática
como un derivado siálico del grupo sanguíneo Lewis A y Pancreatoduodenectomía
se expresa en el 95% de la población. Se ha encontrado Quimioterapia y radioterapia
Drenaje de la vía biliar
una sobreproducción de Ca 19-9 en tumores malignos de
páncreas y del árbol biliar. El tipo de procedimiento quirúrgico a realizar en pacientes
con cáncer de páncreas depende de la localización del
22. Las siguientes aseveraciones son ciertas con tumor. El paciente presenta cáncer de páncreas localizado en
respecto al Ca19.9 en cáncer de páncreas, excepto: cabeza del páncreas. Normalmente el procedimiento de
La elevación de Ca19.9 se presenta hasta en el 80% de los elección en este caso es una pancretoduodenectomía. En
casos de cáncer de páncreas casos específicos (raramente) se tratan con pancretectomía
Niveles de Ca19.9 > 200UI/mL predicen irresecabilidad de la total. Los tumores de cuerpo y cola se tratan con
lesión pancreatecomía distal con esplenectomía.
El Ca19.9 elevado es suficiente para realizar diagnóstico de
cáncer de páncreas 25. La presencia de tromboflebitis migratoria se
La elevación de Ca 19.9 se correlaciona con recurrencia denomina:
durante el seguimiento Síndrome de Trousseau
Signo de Gray-Hunter Recto
Signo de Cullen
Síndrome de Cowden La localización del cáncer de colon por orden de frecuencia
son: sigmoides (35%), ciego (22%), recto (14%), colon
El síndrome de Trousseau se refiere a la presencia de ascendente (12%), colon transverso (10%).
trombosis venosa profunda (flebitis migratoria superficial),
descrita en múltiples neoplasias, de las principales el cáncer
de páncreas en etapas avanzadas. 28. La localización más frecuente de enfermedad
metastásica en el caso de los pacientes con cáncer
de recto es:
CASO CLÍNICO 5 Hígado
Mujer de 55 años, sin antecedentes relevantes. Inició hace 3 Pulmón
meses con disminución del calibre de las heces, además de Hueso
rectorragia en algunas ocasiones. Al interrogatorio dirigido Peritoneo
refiere pérdida de peso de 10kg en 5 meses. A la exploración
física cardiopulmonar y abdomen sin alteraciones, tacto En el caso de cáncer de recto, el pulmón es la localización
rectal se palpa masa de 5 cm en región del recto. más frecuente de metástasis, mientras que en el cáncer de
colon es el hígado en donde se encuentran principalmente
26. Los siguientes son factores de riesgo para las metástasis.
carcinoma colorrectal, excepto:
Historia familiar de cáncer de colon 29. Los siguientes síndromes genéticos se asocian a
Enfermedad inflamatoria intestinal cáncer colorrectal, excepto:
Consumo de antiinflamatorios no esteroideos Poliposis adenomatosa familiar
Dieta alta en grasas y baja en fibra Síndrome de Peutz-Jeghers
Síndrome de Lynch
Síndrome de Trousseau
Los factores de riesgo son: hereditarios (antecedente familiar
de cáncer de colon, poliposis adenomatosa familiar, síndrome Los síndromes genéticos que se asocian con cáncer de colon
de Cowden, Peutz-Jeghers y síndrome de Lynch), tabaquismo, son: poliposis adenomatosa familiar, síndrome de Cowden,
consumo de dieta alta en grasa, consumo de alcohol, Peutz-Jeghers y síndrome de Lynch.
obesidad, enfermedad inflamatoria intestinal. El consumo de
AINEs no se relaciona con el desarrollo de cáncer colorectal. 30. Dentro de los marcadores tumorales útiles durante
el diagnóstico y seguimiento se encuentra:
Ca125
27. La localización anatómica del cáncer de colon más Ca19.9
frecuente es: Antígeno carcinoembrionario
Colon transverso Alfafetoproteína
Colon izquierdo
Sigmoides
El antígeno carcinoembrionario es una de las herramientas
que se usa con mayor frecuencia en el seguimiento del cáncer El carcinoma basocelular es el más frecuente; uno de cada
colorrectal. Ha sido asociado con estadios avanzados, pobre cinco mexicanos lo desarrolla; crece muy lentamente y en el
sobrevida y detección temprana de recurrencia de cáncer 95% de casos no hace metástasis.
colorrectal. Este antígeno, sin embargo, no es específico,
debido a que se encuentra elevado en otras condiciones 3. La lesión precursora del carcinoma epidermoide de piel
malignas, así como en condiciones no neoplásicas, como en la es:
Cicatriz previa
cirrosis hepática, colitis ulcerativa, pancreatitis, e incluso en
Queratosis actínica
fumadores. Léntigo maligno
Linfedema
Cáncer de piel y vías urinarias
La queratosis actínica es la precursora del carcinoma
CASO CLÍNICO 1 epidermoide y muestra la misma atipia queratinocítica, daño
al DNA producida por la radiación UV y mutaciones en p53
Mujer de 70 años, sin antecedentes de importancia. Acude producidas por esta.
por presentar lesión cutánea en dorso nasal no dolorosa de
varios meses de evolución. A la exploración física usted 4. Tratamiento de elección en el cáncer de piel no
detecta una lesión con un borde perlado fino elevado de melanoma:
1cm, con algunas telangiectasias, sin evidencia de Cirugía con bordes quirúrgicos negativos
adenopatías palpables u otras alteraciones. Cirugía con linfadenectomía
Quimioterapia y radioterapia
1. Factor de riesgo más frecuente asociado a cáncer de Vigilancia
piel:
Radiación UV El tratamiento quirúrgico consiste en realizar una extirpación
Edad quirúrgica de la lesión con bordes quirúrgicos negativos.
Sexo femenino Dado que la posibilidad de tener enfermedad metastásica es
Acné muy baja, no se requiere realizar ningún otro procedimiento.

La exposición a la radiación UV causa envejecimiento 5. La variedad más frecuente de carcinoma basocelular es:
prematuro de la piel y daño a la piel que puede resultar en Morfeiforme
cáncer de piel. Pseudoquístico
Nodular
2. El tipo histológico más frecuente de cáncer de piel es: Infiltrativo
Carcinoma espinocelular
Carcinoma basocelular La variedad nodular es la más frecuente del carcinoma
Melanoma maligno basocelular, alrededor del 60% corresponden a esta categoría
Micosis fungoide y se manifiestan por un nódulo eritematoso traslúcido con
telangiectasias.
CASO CLÍNICO 2 La presencia de metástasis en los ganglios linfáticos
regionales es el factor pronóstico más importante en los
Hombre de 60 años, sin otros antecedentes relevantes. pacientes con melanoma, siendo peor el pronóstico a mayor
Refiere lesión hiperpigmentada en región plantar del pie número de ganglios afectados.
derecho de 5 meses de evolución, la cual ha tenido
crecimiento progresivo, con cambios en la pigmentación. A 9. El sitio más frecuente de metástasis es:
la exploración física encuentra lesión de 2cm, de bordes Pulmón
irregulares, hiperpigmentada, además de adenopatía Tejido celular subcutáneo
inguinal ipsilateral. Ganglios linfáticos
Hígado
6. De los siguientes factores de riesgo, uno es
independiente al desarrollo de melanoma: El sitio más frecuente de metástasis es el tejido celular
Fenotipo de piel claro subcutáneo, seguido de los ganglios linfáticos, el pulmón y el
Presencia de nevos comunes hígado.
Radiación ultravioleta
Tabaquismo 10. El índice de Breslow se encarga de medir:
Localización anatómica
El tabaquismo, a pesar de que puede causar cambios Índice mitótico
cutáneos y es factor de riesgo para otras neoplasias, no se ha Profundidad de invasión de acuerdo a las capas de la piel
encontrado como un factor independiente para el desarrollo Profundidad de invasión en milímetros
de melanoma.
La medición del grosor de la lesión en milímetros fue
7. Variedad clínico patológica más frecuente en nuestro descrita en 1970 por Alexandre Breslow, se realiza desde la
país: capa granular de la epidermis hasta el punto más profundo
Melanoma lentigo maligno de la invasión.
Melanoma acral lentiginoso
Melanoma nodular CASO CLÍNICO 3
Melanoma amelánico
Hombre de 65 años, sin antecedentes relevantes. Acude con
En México, a diferencia de otras regiones del mundo, el usted por presentar hematuria macroscópica no dolorosa de
melanoma acral lentiginoso es el tipo más común, con más 4 meses de evolución. Desde hace 2 semanas se agregó dolor
de la mitad de los casos. en flanco derecho. A la exploración física usted encuentra
una tumoración palpable en flanco derecho de
8. El factor pronóstico más importante en melanoma es: aproximadamente 15cm, sin adenopatías u otras
Índice de profundidad alteraciones a la exploración.
Mitosis
Grado histológico 11. El tipo histológico más frecuente del cáncer renal es:
Presencia de ganglios linfáticos con metástasis Papilar
Células claras La triada típica se presenta solo en el 10% de los casos
Cromófobo Los síndromes paraneoplásicos como hipertensión y anemia
Conductos colectores son frecuentes en esta neoplasia

El carcinoma de células claras representa del 60-75% de los La mayor parte de los pacientes se diagnostican en estudios
tumores, seguido por el papilar (15%), cromófobo (5%). radiográficos que se indican por otra razón, usualmente son
tumores pequeños y en etapas iniciales.
12. ¿Cuál no es un factor de riesgo para cáncer renal?
Obesidad 15. El tratamiento en enfermedad localizada es:
Falla renal Cirugía
Edad Quimioembolización
Dieta alta en grasas Radioterapia
Inhibidores de tirosincinasa
Si bien el riesgo de un segundo tumor renal se incrementa en
los pacientes con diagnóstico a edad temprana, este riesgo Si el tumor está localizado, deberá realizarse resección
se incrementa debido a que se piensa que puede tener un quirúrgica, siendo el único tratamiento efectivo para el
componente genético más que porque la edad sea un factor cáncer localizado.
de riesgo. El resto son factores de riesgo conocidos para
cáncer renal. CASO CLÍNICO 4

13. Dentro de los síndromes genéticos que se asocian a Hombre de 65 años, sin comorbilidades. Acude por presentar
cáncer renal se encuentra el siguiente: disminución del chorro urinario, pujo y tenesmo, además de
Síndrome de Gorlin-Goltz nicturia. A la exploración física usted detecta una lesión
Síndrome de Lynch pétrea en lóbulo prostático derecho en el tacto rectal. Se
Síndrome de von Hippel-Lindau realizó un estudio de antígenos prostático específico (APE)
Neoplasia endócrina múltiple tipo 2 con resultado de 8ng/mL.

El síndrome de von Hippel-Lindau está asociado a cáncer 16. El tipo histológico más frecuente de cáncer de próstata
renal hereditario y se relaciona con la histología de células es:
claras, además de quistes renales, pancreáticos, así como Adenocarcinoma ductal
hemangioblastomas cerebelosos, angiomas retinianos y Adenocarcinoma acinar
feocromocitoma. Carcinoma adenoescamoso
Carcinoma con células en anillo de sello
14 Una de las siguientes afirmaciones no es cierta respecto
al cáncer renal: Entre las neoplasias prostáticas malignas, 95% corresponden
La mayor parte de los pacientes se encuentran en etapas a adenocarcinomas acinares, y 5% a carcinomas de células
avanzadas al diagnóstico neuroendocrinas, de células transicionales o sarcomas.
La triada típica de cáncer renal es dolor, hematuria y masa
palpable
17. ¿Cuál de los siguientes, es un factor protector para Se trata de un paciente con enfermedad localizada, de bajo
cáncer de próstata? riesgo por Gleason, por lo que el tratamiento dependerá de
Raza negra la esperanza de vida del paciente y si son buenos candidatos
Historia familiar quirúrgicos. En este caso, al ser un hombre sin
Obesidad comorbilidades, se considera un buen candidato quirúrgico,
Ingesta de isoflavonoides por lo que la prostatectomía radical es el mejor tratamiento
Algunos estudios demuestran cierta acción protectora en para este paciente.
sustancias retinoides presentes en frutas y verduras, así
como vitamina E, micronutrimentos, isoflavonoides, selenio y CASO CLÍNICO 5
vitamina D.
Hombre de 35 años de edad, previamente sano. Refiere
18. Los auxiliares diagnósticos que solicitaría en este caso desde hace 5 meses aumento de volumen a nivel testicular,
es: no doloroso, progresivo. Hace 2 meses se agregó pérdida de
Examen general de orina, cistoscopia peso, además de distensión abdominal y dolor. A la
Ultrasonido transrectal con toma de biopsia exploración física usted documenta adenopatías
Nueva determinación de APE supraclaviculares, además de testículo derecho con aumento
TC de tórax y abdomen de volumen, no dolor, transiluminación negativa.

El diagnóstico presuncional se realiza con la determinación 21. El factor de riesgo con mayor asociación con cáncer de
de APE y la exploración digital rectal. El ultrasonido testículo es:
transrectal permite identificar lesiones sospechosas de Síndrome de Klinefelter
malignidad para tomar una biopsia. Infección por VIH
Criptorquidia
19. El principal sitio de metástasis es: Infertilidad
Pulmón
Ganglios linfáticos El antecedente de criptorquidia es el factor de resigo cuya
Hueso relación es más clara con el desarrollo de un tumor
Sistema nervioso central testicular, estando presente en el 12% de los casos.

El esqueleto, principalmente el axial, es el principal sitio de 22. El tumor testicular más frecuente es:
metástasis en los pacientes con cáncer de próstata. Tumor germinal seminomatoso
Tumor germinal no seminomatoso
20. Se determina que el paciente tiene una enfermedad Carcinoma testicular
localizada, con biopsia con reporte histopatológico Carcinoma epidermoide
Gleason 3+3. La estrategia a seguir con este paciente es:
Quimioterapia y radioterapia El 60% de los tumores germinales son no seminomatosos y se
Resección transuretral de próstata dividen en tumores puros (tumores de senos endodérmicos,
Prostatectomía radical carcinoma embrionario, coriocarcinoma, teratocarcinoma) o
Bloqueo hormonal mixtos.
semanas previas con disnea progresiva, acompañada de
23. Los marcadores tumorales útiles para el diagnóstico plétora facial, red venosa colateral en región torácica. A la
son: exploración física con edema facial, disfonía, disnea de
Alfafetoproteína, antígeno carcinoembrionario y β-HGC reposo, red venosa colateral en tórax anterior, abdomen con
Alfafetoproteína, Ca125 y DHL tumoración palpable en epigastrio y mesogastrio y aumento
β-HGC, alfafetoproteína y DHL de volumen a nivel testicular. Marcadores tumorales con β-
β2 microglobulina, alfafetoproteína y DHL HGC 200,000, DHL 1530 (<170), AFP 10,490 (0-9). Tomografía
con múltiples metástasis pulmonares, hepáticas,
Los marcadores tumorales que se solicitan en caso de adenopatías mediastinales que comprimen la vena cava
sospecha de tumor testicular son alfafetoproteína, fracción β superior, adenopatías retroperitoneales extensas y masa
de la gonadotropina y DHL. testicular.

24. La siguiente aseveración es cierta con respecto a los 26. ¿Qué tipo de tumor presenta el paciente?
marcadores tumorales en cáncer testicular: Tumor germinal seminomatoso
La DHL es inespecífica, por lo que no es útil dentro de los Tumor germinal no seminomatoso
marcadores tumorales Sarcoma retroperitoneal
La β-HGC se eleva únicamente en tumores germinales no Carcinoma de primario desconocido
seminomatosos
La alfafetoproteína se eleva únicamente en tumores En este caso, el paciente tiene marcadores tumorales
germinales no seminomatosos claramente positivos, por lo que se realiza el diagnóstico de
La β-HGC se produce en el sincitiotrofoblasto tumor germinal, dado que tiene elevada la alfafetoproteína,
En los tumores germinales, la alfafetoproteína se restringe a el diagnóstico es de un tumor germinal no seminomatoso.
los tumores germinales no seminomatoso. 27. La sintomatología respiratoria está causada por:
Tromboembolia pulmonar
25. El manejo inicial de este paciente debe ser: Metástasis pulmonares
Biopsia testicular Síndrome de vena cava superior
Orquiectomía radical Taponamiento cardiaco
Quimioterapia
Hormonoterapia El síndrome de vena cava superior se caracteriza por disnea
progresiva, red venosa colateral, edema facial, plétora
El tratamiento inicial de un tumor testicular es la facial, y disfonía y se presenta en tumores germinales.
orquiectomía radical inguinal, que además brinda un
diagnóstico definitivo. 28. ¿Qué aseveración no corresponde con el caso clínico?
Se requiere forzosamente biopsia testicular para confirmar el
CASO CLÍNICO 6 diagnóstico
La presencia del cuadro clínico y los marcadores tumorales
Hombre de 35 años, sin antecedentes relevantes. Acude por elevados justifican el inicio de tratamiento
presentar masa testicular no dolorosa de 1 año de evolución, Las características clínicas corresponden a un tumor de
posteriormente se agregó lumbalgia desde hace 4 meses y 2 pobre pronóstico
La mayor frecuencia de tumores no seminomatosos es entre una radiografía de tórax donde se observa una masa apical
los 15-35 años izquierda de 6cm que se extiende hasta el mediastino.

La presencia del cuadro clínico y los marcadores tumorales 1. Su sospecha diagnóstica es:
elevados justifican el inicio del tratamiento, y dado los Tuberculosis pulmonar
marcadores, no es necesario forzosamente realizar una Metástasis pulmonares
biopsia testicular para confirmar el diagnóstico. Cáncer de pulmón
Tromboembolia pulmonar
29. El siguiente síndrome genético se asocia a tumores
germinales primarios mediastinales: En este caso el paciente cuenta con factores de riesgo,
Síndrome de Turner además de datos clínicos de síndrome de vena cava superior
Síndrome de feminización testicular y síndrome de Horner, en la radiografía se encuentra una
Síndrome de Klinefelter masa apical, por lo que la sospecha debe ser malignidad.
Síndrome de Down
2. Para confirmar su sospecha diagnóstica usted solicita los
El síndrome de Klinefelter se asocia con la presencia de siguientes estudios:
tumores germinales, principalmente de localización BAAR en expectoración y PPD
mediastinal. TC de tórax y abdomen, con toma de biopsia de la lesión
Angiotomografía de arterias pulmonares
30. La edad de presentación más frecuente de los tumores Es un diagnóstico clínico
germinales es:
Entre los 40-60 años El estudio inicial recomendado es la TC de tórax y abdomen
Entre los 30-50 años superior, que permite evaluar las características y extensión
Entre los 15-35 años del tumor primario, sospechar si existe daño ganglionar
Mayores de 60 años mediastinal y puede confirmar o descartar la presencia de
metástasis hepáticas o suprarrenales. Posteriormente se
El mayor pico de incidencia de los tumores germinales es realiza una biopsia de la lesión para confirmar el tipo
entre los 15 y los 35 años de edad en los hombres. histológico.

Misceláneos 3. La triada de miosis, ptosis y anhidrosis se denomina:


Síndrome de Trousseau
CASO CLÍNICO 1 Síndrome de Horner
Síndrome de Holter
Hombre de 75 años, acude por presentar disnea, plétora Síndrome de vena cava superior
yugular y edema facial. Tiene antecedente de tabaquismo El síndrome de Horner se caracteriza por miosis, ptosis y
intenso desde los 20 años, hasta 2 cajetillas al día. A la anhidrosis, además de la presencia de enoftalmos. Se da por
exploración física usted encuentra miosis, ptosis y anhidrosis afección de la cadena simpática, en este caso por infiltración
de hemicara derecha, además de edema facial y en ambos tumoral.
brazos, red venosa colateral en tórax anterior. Se solicita
4. La presencia de edema facial, red venosa colateral y 6. Por el cuadro clínico y las características de la paciente,
plétora yugular lo hacen sospechar de: usted piensa que en el reporte histopatológico usted
Síndrome de compresión medular obtendría el siguiente resultado:
Síndrome de Horner Cáncer de pulmón células no pequeñas variedad
Síndrome de vena cava superior adenocarcinoma con mutación EGFR negativa
Tamponade cardiaco Cáncer de pulmón células no pequeñas variedad
adenocarcinoma con mutación EGFR positiva
Todos estos son características clínicas del síndrome de vena Cáncer de pulmón células no pequeñas variedad epidermoide
cava superior, siendo los tumores más frecuentes que lo con mutación EGFR negativa
provocan los linfomas, los tumores germinales y el cáncer de Cáncer de pulmón de células pequeñas
pulmón.
Dentro de las características clínicas para sospechar
5. El tipo histológico más frecuente de cáncer de pulmón presencia de mutación EGFR, se encuentra edad menor de 50
es: años, mujeres, sin historia de tabaquismo. En México, otro
Cáncer de células no pequeñas variedad adenocarcinoma factor que se ha asociado a presencia de mutación EGFR es la
Cáncer de células no pequeñas variedad epidermoide presencia de exposición al humo de leña.
Cáncer de células grandes
Carcinoma bronquiolo alveolar 7. Dentro de los factores de riesgo en general para cáncer
de pulmón, el más importante es:
A lo largo de los últimos años se ha observado una transición Tabaquismo
en los tipos histológicos del cáncer de pulmón de células no Exposición a radiación
pequeñas, siendo más frecuente la variedad histológica del Exposición a humo de leña
adenocarcinoma. Exposición a radón

CASO CLÍNICO 2 El tabaquismo es el principal factor de riesgo de esta


neoplasia y se estima que hasta 90% de los casos se deben al
Mujer de 40 años acude a la consulta por presentar disnea consumo de tabaco.
progresiva, llegando a ser hasta de pequeños esfuerzos,
pérdida de peso de 7kg y hace 2 semanas se agregó 8. El cáncer de pulmón representa la siguiente causa de
expectoración con algunos rastros hemáticos. Antecedente muerte por cáncer en hombres:
personal no patológico: exposición al humo de leña desde la 1
infancia. A la exploración física usted documenta 2
adenopatías supraclaviculares bilaterales y síndrome de 3
derrame pleural del 50%. Indica realización de tomografía de 4
tórax y abdomen, donde se documenta la presencia de una
lesión en pulmón derecho, periférica, de 5cm, acompañada De acuerdo al INEGI, el cáncer de pulmón representa la
de derrame pleural ipsilateral, adenopatías cervicales, segunda causa de muerte en hombres por cáncer, sólo
mediastinales y retroperitoneales, metástasis suprarrenales, superado por el cáncer de próstata.
hepáticas.

9. El tratamiento de una paciente con enfermedad Solicitar tiroglobulina


metastásica como este caso consiste en: Solicitar ultrasonido tiroideo y BAAF
Cirugía Vigilancia con nuevas pruebas de función tiroidea en 3 meses
Quimioterapia paliativa Referir inmediatamente para realizar tiroidectomía
Quimiorradioterapia
Colocación de sonda pleural a permanencia Ante la presencia de un nódulo tiroideo en un paciente
eutiroideo, lo que se recomienda es realizar un ultrasonido
El tratamiento de la enfermedad metastásica de forma con una biopsia por aspiración con aguja fina.
inicial es quimioterapia paliativa, que ha mostrado mejoría
en la calidad de vida y beneficio en supervivencia. En este 12. El tipo histológico más frecuente de cáncer de tiroides
caso asumimos que no se conoce la mutación de EGFR. De es:
conocerse que está ausente la mutación o que no se puede Papilar
obtener, el tratamiento debe ser quimioterapia. Folicular
Medular
10. En caso de confirmarse mutación de EGFR presente, el Linfoma
tratamiento de esta paciente sería:
Quimioterapia paliativa El carcinoma papilar representa del 75-85% de los tumores
Inhibidor de tirosincinasa de EGFR tiroideos, seguido por el carcinoma folicular (10-15%).
Radioterapia
Quimiorradioterapia 13. El factor pronóstico más importante es:
Metástasis ganglionares
Los inhibidores de EGFR han demostrado mejorar la Grado histológico
supervivencia global de los pacientes con mutaciones de Edad
EGFR, entre ellos se encuentran erlotinib, gefitinib, afatinib, Metástasis a distancia
entre otros, por lo que de conocerse que la mutación está
presente, deben ser la primera línea de tratamiento de estos El factor pronóstico más importante es la edad, lo pacientes
pacientes. después de los 40-45 años presentan mayor agresividad local
y mayor mortalidad.
CASO CLÍNICO 3
14. El sitio más frecuente de metástasis es:
Mujer de 40 años, acude por presentar aumento de volumen Sistema nervioso central
en región cervical anterior de 5 meses de evolución, Pulmón
progresiva, no dolorosa. A la exploración física usted palpa Tejidos blandos
aumento del lóbulo tiroideo derecho a expensas de nódulo Hígado
tiroideo de 3cm, además de adenopatías cervicales
ipsilaterales. Usted solicita pruebas de función tiroidea, las Los sitios más frecuentes de metástasis son el pulmón (50%),
cuales son normales. hueso (25%), sistema nervioso central y otros tejidos blandos
(10%).
11. La conducta a seguir es la siguiente:
15. El manejo de esta paciente, si se confirma diagnóstico La evaluación de tumores de tiroides medulares debe incluir
de malignidad debe ser: medición de calcitonina, que siempre se encuentra elevada.
Administración de I131
Cirugía 18. Esta entidad se asocia a mutaciones en el siguiente
Quimioterapia gen:
Quimiorradioterapia MET
RET
El tratamiento óptimo del cáncer de tiroides es la cirugía. La HER2
tiroidectomía total o casi total debe realizarse en los Transportador de yodo
pacientes con cáncer diferenciado con tumores mayores de
1cm, mayores de 45 años, con enfermedad regional. Si están Mutaciones del gen RET se asocian a cáncer medular de
negativos estos factores de riesgo, y son menores a 1cm sin tiroides, ya sean mutaciones de novo o asociadas a síndromes
metástasis, se puede considerar la hemitiroidectomía. genéticos como MEN2.

CASO CLÍNICO 4 19. El tratamiento de esta paciente debe ser:


Tiroidectomía total
Mujer de 44 años, previamente sana. Acude por presentar Resección únicamente del nódulo tiroideo
nódulo en cara anterior de cuello de 3cm de 5 meses de Quimioterapia y radioterapia
evolución. Asintomática. Se le tomó una biopsia con reporte Terapia con I 131
de neoplasia derivada de células parafoliculares (células C).
El único tratamiento con posibilidad de curación es la
16. El reporte histopatológico es consistente con: cirugía, que es la tiroidectomía total con disección central
Cáncer papilar de tiroides más disección ipsilateral, si se sospecha enfermedad
Cáncer folicular de tiroides ganglionar extensa se recomienda que la disección sea
Cáncer medular de tiroides bilateral.
Cáncer anaplásico de tiroides
20. Dentro de los casos hereditarios, los más frecuentes
El carcinoma medular es una entidad rara derivada de las son asociados a:
células C o parafoliculares que se caracteriza por secretar Neoplasia endócrina múltiple tipo 2A
calcitonina y corresponde al 5% de las neoplasias tiroideas. Neoplasia endócrina múltiple tipo 2B
Cáncer de tiroides hereditario
17. Dentro de los marcadores tumorales útiles en esta Tiroiditis de Hashimoto
neoplasia, se encuentra:
Tiroglobulina Los pacientes con MEN2A representan 80% de los casos
Calcitonina hereditarios, desarrollan liquen plano cutáneo, amiloidosis,
Alfafetoproteína feocromocitoma.

No existen marcadores tumorales útiles CASO CLÍNICO 5


Hombre de 50 años, acude con usted por presentar aumento La vía hematógena es la forma más común de diseminación a
de volumen en extremidad inferior derecha, que ha distancia, siendo el pulmón el sitio más afectado (35%),
aumentado de forma progresiva, no dolorosa, que limita la seguido del hígado, hueso y cerebro.
deambulación. A la exploración física encuentra un tumor en
muslo derecho de 10cm, duro, fijo a planos profundos, no 24. Esta patología no está asociada como factor de riesgo
doloroso. Le realizaron tomografía donde se evidencia lesión para sarcomas:
dependiente de tejidos blandos, de 10cm. Se tomó biopsia Síndrome de Gardner
con reporte histopatológico de sarcoma. Síndrome de Li-Fraumeni
Neurofibromatosis tipo 1
21. El tipo histológico más frecuente de sarcoma de Síndrome de cáncer de mama-ovario
extremidades es:
Leiomiosarcoma Las mutaciones en BRCA no se han asociado de forma
Histiocitoma fibroso maligno consistente a mayor riesgo de sarcomas.
Sarcoma sinovial
Rabdomiosarcoma 25. El tratamiento inicial de este paciente debe ser:
Quimioterapia
En las extremidades, el tipo histológico más frecuente es el Radioterapia
histiocitoma fibroso maligno (30%), seguido de liposarcoma y Resección amplia de la lesión
leiomiosarcoma. Amputación supracondílea

22. La localización más frecuente de los sarcomas de El tratamiento primario de los sarcomas de partes blandas es
tejidos blandos es: la resección con un margen tridimensional negativo.
Extremidades
Retroperitoneo CASO CLÍNICO 6
Abdomen
Cabeza y cuello Hombre de 20 años, acude por presentar aumento de
volumen en la rodilla derecha, de 6 meses de evolución,
Según su localización, la mayoría se originan en doloroso, acompañado de pérdida de peso de 10kg. A la
extremidades (59%), tronco (18%), retroperitoneo (13%) y exploración física, se documenta aumento de volumen en
cabeza y cuello (9%). rodilla derecha con dificultad para realizar arcos de
movimiento en rodilla derecha. Se tomó radiografía de la
23. El sitio más frecuente de metástasis es: extremidad con hallazgo de lesión perióstica con signo del
Hueso sol naciente.
Cerebro
Pulmón 26. El tumor óseo más frecuente después del mieloma es:
Ganglios Osteosarcoma
Sarcoma de tejidos blandos
Condrosarcoma
Tumor de células gigantes
Amputación de la extremidad
Después del mieloma, el osteosarcoma es el tumor óseo Sólo cirugía
primario más frecuente. Sólo quimioterapia

27. El sitio anatómico más frecuentemente afectado en los El tratamiento de los sarcomas óseos debe ser
osteosarcomas es: multidisciplinario e incluye quimioterapia.
Extremo distal del fémur
Extremo proximal del fémur
Extremo proximal de la tibia EXAMEN FINAL
Cráneo
CASO CLÍNICO 1
El osteosarcoma puede afectar a cualquier hueso, pero se
localiza preferentemente en las metáfisis de los huesos Mujer de 65 años, con antecedente de obesidad mórbida.
largos, se ubica principalmente en la rodilla, siendo el Antecedentes ginecológicos: Gesta 1, Parto 1; cursó con
extremo distal del fémur (40%) la localización más frecuente. problemas de fertilidad, requirió uso de hormonales orales.
Acude a su consulta por presentar sangrado transvaginal
28. La mutación asociada al sarcoma de Ewing es la escaso. A la exploración física usted no encuentra algo
siguiente: relevante.
11; 22
8; 14 1. ¿Cuál de los siguientes no es un factor de riesgo
9; 18 para neoplasias endometriales?
14; 18 Diabetes mellitus
Hipertensión arterial
Del 85-90% de los tumores de Ewing presentan la Multiparidad
translocación t (11; 22) (q24; q12) que forma una proteína de Uso de tamoxifeno
fusión anormal denominada EWS-FLI.
Los factores de riesgo para cáncer de endometrio son los
29. El sitio principal de metástasis en los sarcomas óseos relacionados a exposición de estrógenos: obesidad,
es nuliparidad, diabetes, hipertensión, antecedentes familiares
Ganglios y antecedente de irradiación pélvica.
Hueso
Pulmón 2. El estudio que usted indica a su paciente es:
Tejidos blandos Ultrasonido transvaginal y toma de biopsia
Resonancia magnética de pelvis
Hasta el 80% de las metástasis de los sarcomas óseos son al Exploración abdominal
pulmón. Tomografía de tórax y abdomen

30. El tratamiento de este paciente debe ser: Ante la sospecha de cáncer de endometrio, la exploración
Multimodal con quimioterapia, cirugía y radioterapia pélvica y la especuloscopía pueden mostrar extensión a
cuello o vagina. Sin embargo, ante la sospecha se debe Hiperplasia endometrial
realizar un estudio histopatológico de la cavidad Endometriosis
endometrial. El ultrasonido es útil para determinar la Endometrioma
presencia de engrosamiento endometrial como abordaje Miomatosis uterina
diagnóstico de sangrado uterino anormal.
La hiperplasia endometrial se ha identificado como una
3. La siguiente afirmación es cierta con respecto al lesión premaligna. Cuando existe atipia, la malignización
cáncer endometrial: ocurre entre 8-29% de los casos con hiperplasia simple y
La presentación más frecuente es dolor pélvico compleja, respectivamente.
La edad de presentación es en menores de 50 años CASO CLÍNICO 2
El 75% se diagnostica en etapas tempranas
Un endometrio de >1cm puede vigilarse en 6 meses Hombre de 55 años, acude con usted por pérdida de peso de
10kg en 3 meses y dolor abdominal en fosa iliaca izquierda,
La presentación clínica más frecuente es sangrado uterino además de notar desde hace 3 meses episodios de
anormal, a menudo en mujeres postmenopáusicas: La edad rectorragia. Tiene antecedente de consumo de tabaco y
de presentación es después de la sexta década de la vida. El alcohol ocasional. Niega enfermedades crónico-
tipo histológico frecuente es el carcinoma endometrioide degenerativas. A la exploración física: palidez de
(60-65%). En cualquier mujer que se detecte engrosamiento tegumentos, signos vitales normales, cardiopulmonar sin
endometrial >1cm requiere evaluación y estudio compromiso, abdomen no se palpan megalias. Usted realiza
histopatológico para descartar lesiones premalignas o tacto rectal sólo con huellas de sangre, no se palpa tumor.
malignas aunque el 75% de los diagnósticos son en etapas
tempranas. 6. ¿Cuál sería su conducta a seguir?
Realizar tomografía de tórax y abdomen como abordaje de
4. El tratamiento inicial de esta paciente es el pérdida de peso
siguiente: Tratamiento con antiespasmódicos y dieta alta en fibra
Legrado uterino Realizar biometría hemática y colonoscopia
Cirugía Pedir coproparasitoscópico y coprocultivo por sospecha de
Radioterapia externa amibiasis
Braquiterapia
En el caso de este paciente con síntomas gastrointestinales y
La cirugía es útil para evaluar la extensión de la enfermedad sangrado de tubo digestivo, además de datos de alarma
y la estadificación además es terapéutica, en las pacientes (pérdida de peso), dado el grupo de edad (>50 años), debe
con enfermedad confinada al útero, es la parte inicial y descartarse una neoplasia gastrointestinal como causa de la
fundamental del tratamiento. En caso de identificarse sintomatología. Por la presencia de sangrado se deberá
factores que indiquen riesgo de recaída se adiciona descartar la presencia de anemia. Si la sospecha es de cáncer
quimioterapia o radioterapia adyuvante. colorrectal, la colonoscopia es el estudio de elección, con lo
que se puede visualizar completo el colon y recto, tomar
5. ¿Cuál de las siguientes se considera una lesión biopsias del tumor y detectar tumores sincrónicos.
premaligna endometrial?
7. ¿Cuál de los siguientes NO es un factor de riesgo Ca 19.9 se eleva en otras neoplasias, principalmente
para cáncer de colon? pancreatobiliares.
Poliposis adenomatosa familiar
Enfermedad inflamatoria intestinal 10.Sus estudios de extensión no muestran enfermedad
Síndrome de Peutz-Jeghers a distancia. ¿Qué tratamiento requiere este
Síndrome de Li-Fraumeni paciente?
Radioterapia abdominal
Los síndromes genéticos que se asocian con cáncer de colon Cirugía
son: poliposis adenomatosa familiar, síndrome de Cowden, Resección endoscópica de la lesión
Peutz-Jeghers y síndrome de Lynch. El síndrome de Li- Colostomía paliativa
Fraumeni se encuentra asociado a mutaciones en p53 y
dentro de los tumores asociados se encuentra cáncer de El tratamiento oncológico en el caso de cáncer de colon es
mama y sarcomas. multidisciplinario, pero la intervención quirúrgica tiene una
participación fundamental en el pronóstico. El tratamiento
8. Se realiza colonoscopia con hallazgo de lesión inicial es la resección quirúrgica, y de acuerdo a las
exofítica de 8 cm a nivel de sigmoides, friable, que características patológicas se decide tratamiento adyuvante,
ocupa el 80% de la luz. Se toman biopsias con en general con quimioterapia.
reporte de malignidad. ¿Cuál es el tipo histológico
más frecuente? CASO CLÍNICO 3
Tumor del estroma gastrointestinal
Adenocarcinoma Hombre de 75 años, cuenta con antecedente de consumo de
Carcinoma epitelioide tabaco y alcohol intenso durante 40 años. Ha presentado
Linfoma ictericia por 6 meses, con coluria y acolia, dolor abdominal
en epigastrio y pérdida de peso de 10kg en 3 meses. Acude el
Hasta 98% de las neoplasias malignas corresponden a día de hoy por aumento de volumen en extremidad inferior
adenocarcinomas. Otras neoplasias raras son carcinoide, izquierda, con dolor, edema y eritema. Exploración física:
sarcoma, linfoma y tumor del estroma gastrointestinal. Conjuntivas ictéricas, ganglio supraclavicular derecho,
abdomen con palpación de la vesícula en hipocondrio
9. ¿Cuáles son los estudios de extensión que se derecho, no se palpa hepato o esplenomegalia. Laboratorios
recomienda realizar en este paciente? con BT 21, BD 17, BI 4, ALT 55, AST 33, FA 455.
Medición de antígeno carcinoembrionario (ACE)
Ultrasonido abdominal y radiografía de tórax 11.Su principal sospecha diagnóstica es:
Tomografía de tórax-abdomen y medición de ACE Coledocolitiasis
Gammagrama óseo Hepatitis viral
Cáncer de páncreas
Se recomienda imagen del tórax (Radiografía de tórax o Pancreatitis crónica
tomografía), tomografía de abdomen y pelvis y
determinación de antígeno carcinoembrionario en sangre. El Dentro de los datos clínicos que sugieren la presencia de
cáncer de páncreas, se encuentra la edad, el antecedente de
tabaquismo, ictericia, pérdida de peso e hiperbilirrubinemia Signo de Courvoisier
a expensas de bilirrubina directa con síndrome colestásico. Signo de Virchow
Además, este paciente presenta el signo de Courvoisier Síndrome de Trousseau
(vesícula biliar palpable), asociado con neoplasias Síndrome de Lynch
pancreatobiliares en el 25% de los pacientes.
El síndrome de Trousseau se refiere a la presencia de
12.El estudio que solicita para confirmar su diagnóstico trombosis venosa profunda (flebitis migratoria superficial),
es: descrita en múltiples neoplasias, de las principales el cáncer
Tomografía trifásica del páncreas y Ca 19.9 de páncreas
Colangiografía pancreática endoscópica
Laparoscopía diagnóstica 15.El tratamiento de este paciente es:
Perfil de hepatitis viral Radioterapia y colocación de prótesis biliar
Derivación percutánea de la vía biliar
Para obtener imágenes óptimas en el diagnóstico, se utiliza Cirugía y quimioterapia adyuvante
la TAC trifásica, esto permite distinguir la masa pancreática y Quimioterapia y colocación de prótesis biliar
evalúa los vasos peripancreáticos para evaluar resecabilidad.
El Ca19.9 se eleva en 80% de los casos. El tratamiento de la enfermedad metastásica es paliativo. No
tienen papel el tratamiento quirúrgico o con radioterapia a
13.Se confirma el diagnóstico de cáncer de páncreas, excepción de medidas paliativas. En este paciente que
con presencia de metástasis hepáticas, peritoneales presenta ictericia obstructiva se debe realizar como
y ganglionares. La siguiente aseveración es cierta paliación colocación de prótesis biliar, inicialmente por
con respecto al cáncer de páncreas: métodos endoscópicos y solamente de ser fallida valorar
La mayor parte del diagnóstico es en menores de 50 años colocación de drenajes externos.
El tipo histológico más frecuente es el adenocarcinoma
ductal CASO CLÍNICO 4
La localización más frecuente es en la cola del páncreas
La mayor parte son resecables al diagnóstico Mujer de 65 años, sin enfermedades previas. Acude porque
se palpó masa en fosa iliaca izquierda, la cual es
El 85% de los casos se encuentran entre los 60-80 años, con ligeramente dolorosa, niega otra sintomatología.
una mediana de 69 años, es más frecuente en el sexo Antecedentes ginecoobstétricos: 1 gesta, 1 parto, fecha de
masculino con una proporción 1.5:1. 65% de los tumores se última menstruación hace 4 años. Utilizó anticonceptivos
ubican en la cabeza del páncreas, 20% en el cuerpo o cola y orales por 5 años. Acudió con otro médico quien le realizó
15% son difusos. Al diagnóstico, sólo 15% de los pacientes son ultrasonido transvaginal, trae el reporte que indica el
candidatos a cirugía, 40% tiene enfermedad irresecable y 45% hallazgo de lesión ovárica izquierda septada, con algunas
tiene metástasis al diagnóstico. El 85% de los tumores áreas sólidas, hipervascular, de 6cm; no se observó líquido
malignos corresponden a adenocarcinomas ductales. libre.
16.¿Cuál es su sospecha diagnóstica?
14.El cuadro clínico de trombosis venosa profunda Enfermedad pélvica inflamatoria
asociada a tumor pancreático se denomina: Absceso tubo-ovárico
Cáncer de ovario
Trastorno funcional digestivo 19.Una de las siguientes aseveraciones es cierta con
respecto al cáncer de ovario:
To d a m u j e r c o n u n a m a s a a n e x i a l e n m u j e r e s El 75% se diagnostica en etapas tempranas
postmenopáusicas, tumor sólido al ultrasonido mayor a 8 cm El uso de anticonceptivos orales se considera uno de los
debe ser sospechosos de malignidad. factores de riesgo más fuertes
Dentro de los factores de riesgo se encuentran la infertilidad
17.¿Qué estudios le pueden ayudar a corroborar su y la menopausia tardía
sospecha diagnóstica? Un Ca125 elevado es diagnóstico de cáncer de ovario
Cultivo de secreción cervicovaginal
Prueba terapéutica con anticonceptivos orales Dentro de los factores de riesgo se encuentra la historia
Toma de biopsia y determinación de Ca125 familiar, la edad >50 años, menarca temprana, menopausia
Colonoscopia tardía, infertilidad. El uso de anticonceptivos orales se
considera un factor protector. La elevación del Ca125 es útil,
El diagnóstico histológico se realiza mediante biopsia sin embargo, no es diagnóstica, ya que se puede elevar en
percutánea, abierta o por vía laparoscópica con el fin de enfermedades que cursan con inflamación del peritoneo y en
obtener una muestra adecuada del tumor y realizar una otros tumores como cáncer de mama o pulmonar.
estadificación quirúrgica. No debe realizarse ruptura de la
cápsula del tumor por el riesgo de diseminación abdominal. 20.De los síndromes hereditarios asociados a cáncer, el
El Ca125 se encuentra elevado hasta en el 85% de los casos más asociado a cáncer de ovario es:
cuando se emplea el límite de corte de 35 UI. Trousseau
Lynch
18.Se confirma su sospecha diagnóstica. El tratamiento BRCA 1 y 2
inicial de esta paciente debe ser: Peutz-Jeghers
Tratamiento quirúrgico buscando citorreducción óptima
(histerectomía, salpingooforectomía bilateral, Sólo 5-10% de los pacientes con cáncer de ovario son
omentectomía, biopsia de peritoneo, lavado peritoneal) hereditarios. Las mutaciones de BRCA 1 y 2 tienen un riesgo
Tratamiento quirúrgico sólo quitando la lesión sospechosa de cáncer de ovario de 15-60%.
Tratamiento quirúrgico con drenaje del absceso
Tratamiento antibiótico CASO CLÍNICO 5

La cirugía es el estándar de oro en la estadificación; el Hombre de 25 años, estudiante. No tiene antecedentes de


objetivo de la cirugía es retirar toda la enfermedad y importancia. Inició hace 8 meses con masa testicular no
determinar el estadio clínico. La rutina de ovario debe incluir dolorosa, posteriormente refiere lumbalgia y en los últimos
exploración sistemática de la cavidad peritoneal, 15 días se ha agregado disnea. A la exploración física con
histerectomía total y salpingooforectomía bilateral, signos vitales normales, sin adenopatías, en abdomen se
omentectomía inframesocólica, biopsias de correderas palpa tumoración de 10cm en mesogastrio, además de lesión
parietocólicas, fondo de saco y peritoneo visceral, ambos testicular pétrea en testículo izquierdo de 5cm, no dolorosa.
hemidiafragmas y linfadenectomía. Transiluminación negativa.
alfafetoproteína se restringe a los tumores germinales no
21.¿Cuál es su sospecha diagnóstica? seminomatosos.
Orquiepididimitis crónica
Cáncer de testículo 24.Usted realiza una tomografía, donde se observa un
Tuberculosis genital conglomerado retroperitoneal de 5cm, sin
Quiste de epidídimo metástasis pulmonares o hepáticas. La conducta a
seguir es la siguiente:
Se trata de un hombre que se encuentra dentro de la edad Iniciar quimioterapia
de riesgo para cáncer testicular, cuya manifestación más Administrar radioterapia al retroperitoneo
común es una masa escrotal indolora. Realizar orquiectomía radical
Realizar biopsia de la lesión retroperitoneal
22.¿Qué estudios solicita para confirmar su sospecha
diagnóstica? El tratamiento inicial y el diagnóstico definitivo se
Ultrasonido testicular y marcadores tumorales establecen a través de una orquiectomía radical.
Urocultivo Posteriormente, de acuerdo a la etapa clínica, se realizará la
PCR para tuberculosis en orina elección del tratamiento adyuvante. Sólo los pacientes que
Se puede establecer clínicamente el diagnóstico se presenten con una urgencia oncológica pueden iniciar con
tratamiento con quimioterapia si se confirma elevación de
En el caso del abordaje diagnóstico, la ultrasonografía marcadores tumorales y cuadro clínico sugestivo de tumor
testicular permite distinguir una masa, algunas germinal.
calcificaciones. Deben cuantificarse siempre los marcadores
tumorales. Si no hay confirmación histológica, los 25.Posterior al tratamiento que eligió, la siguiente fase
marcadores muy elevados son diagnósticos, principalmente es:
en tumores no seminomatosos. Iniciar quimioterapia
Administrar radioterapia al retroperitoneo
23.El ultrasonido testicular muestra una lesión de 5cm Realizar orquiectomía radical
con calcificaciones e hipervascular. Los marcadores Tomar biopsia del testículo
tumorales que realiza con DHL 915 (0-150), AFP 75
(0-9), BHGC 255 (0-3). Con estos resultados usted: Se trata de un paciente con un tumor germinal no
Establece el diagnóstico de tumor germinal seminomatoso seminomatoso con enfermedad retroperitoneal >5cm (Etapa
Establece el diagnóstico de tumor germinal no seminomatoso clínica II). Posterior a la orquiectomía radical estos pacientes
Decide vigilancia estrecha requieren quimioterapia basada en platino.
Repite los estudios de laboratorio

Las características de los estudios de imagen que nos hacen CLINICAL CASE 6
sospechar de una neoplasia testicular son tumores sólidos
con calcificaciones múltiples y difusas. Dentro de la A 68-year old man is brought to the emergency room after
elevación de los marcadores tumorales, la elevación de coughing up a cupful of red blood. The previous 3-4 months,
he had presented a chronic non-productive cough, but no
fever. More recently, he noticed some blood-streaked Antibiotics and reassurance of the lung mass
sputum. On review of his symptoms, he reports strenuous Pulmonary function test to evaluate pulmonary reserve to
fatigue and 15kg weight loss in the past 3 months. The evaluate for pneumonectomy
patient has smoked one pack of cigarettes per day for the Obtain a specific tissue diagnosis by biopsy of the hilum mass
past 35 years. Physical examination: No lymphadenopathy, Pleurodesis
chest with scattered rhonchi bilaterally. No other relevant
findings. Es imperativo que cuando se encuentra una masa pulmonar
se debe intentar obtener diagnóstico histológico para
26.Which one is your most likely diagnosis? determinar el tratamiento. El objetivo primario es
Tuberculosis determinar el tipo histológico y si el paciente es candidato a
Lung cancer cirugía, establecer la extensión de la enfermedad.
Community-acquired pneumonia
Chronic obstructive pulmonary disease 29.In chest CT you find a hilum 7-cm lung mass with
ipsilateral pleural effusion (30%). According to
Este paciente tiene como factor de riesgo un índice tumor location, which histologic type would you
tabáquico elevado, además de síntomas como tos no expect to find in lung biopsy?
productiva, fiebre y posteriormente esputo sanguinolento y Small cell lung cancer
pérdida de peso, por lo que dentro de su abordaje de la Adenocarcinoma
hemoptisis deberá descartarse cáncer pulmonar. Squamous-cell carcinoma
Lymphoma
27.What is your next step to confirm your presumptive
diagnosis? Los carcinomas escamosos se relacionan más con una
Obtain complete blood count and chest CT localización central, con hombres fumadores y de alta edad.
Prescribe antibiotics for pneumonia
Mantoux test and bacilloscopy 30.Which one is the leading risk factor for lung cancer?
Pulmonary-function test and bronchodilators Alcohol consumption
Occupational agents (asbestos)
Dado que la sospecha es cáncer pulmonar, el diagnóstico se Cigarette smoking
debe obtener un estudio de imagen. El estudio inicial Air pollution
recomendado es TC de tórax y abdomen superior, que
permite evaluar características y extensión de tumor Aunque el cáncer de pulmón es una enfermedad
primario y evaluar si existen ganglios mediastinales y multifactorial, el tabaquismo representa el principal factor
metástasis. En este caso particular, se requiere biometría de riesgo para esta neoplasia y hasta 90% de los casos de
hemática para evaluar si el sangrado tiene repercusión cáncer de pulmón se deben a consumo de tabaco.
(anemia).
CLINICAL CASE 7
28.In chest CT you find a hilum 7-cm lung mass with
ipsilateral pleural effusion (30%). What is the best A 57-year-old postmenopausal woman presents for annual
next step? examination. She noticed 3 months ago a lump in her right
breast. No other symptoms. Physical examination: 3cm non-
tender, movable, irregular lump, in right breast, no skin El tratamiento de la enfermedad temprana (I y II) es
changes, no nipple discharge. No axillary lymphadenopathy. quirúrgico, el abordaje incluye la escisión de todo el tejido
NO other significant findings. involucrado y el manejo de la axila. En cuanto al manejo de
31.What will be your next step in this patient? la axila, existen varios estudios que han mostrado la utilidad
Breast ultrasound del ganglio centinela en mujeres con tumores T1-T2 y
Mammogram ganglios clínicamente negativos. Si el ganglio centinela es
Start hormonal therapy negativo, no está indicada la disección axilar.
Aspiration of cyst
34.The most important prognostic factor in the
Ante la presencia de una masa palpable en una paciente treatment of this disease is:
postmenopáusica, la mastografía es el mejor método de Size of tumor
detección para cáncer de mama con sensibilidad de 70-75%. Axillary metastases
El ultrasonido es un complemento esencial, y es útil en la Estrogen receptors of the tumor
identificación de lesiones en mamas densas. HER2 expression of the tumor

32.In mammogram you find a spiculated 3-cm nodule El principal factor pronóstico clínico es el estado ganglionar.
with no lymphadenopathy. A biopsy was performed El pronóstico guarda relación directa con el número de
with grade 3 ductal carcinoma of the breast, ER ganglios afectados. Otros factores pronósticos son el tamaño
positive, PR positive and HER2 negative. According de tumor, edad al diagnóstico, invasión linfovascular,
to TNM staging system, this patient would be: proliferación celular, expresión de receptores hormonales y
T1N0M0 HER2.
T2N0M0
T3N0M0 35.In Mexico, the recommended mammogram
T3N1M0 screening is:
From 40 to 69 years old, annually
De acuerdo a la estadificación TNM, esta paciente tiene un From 40 to 69 years old, every 2 years
tumor de 3cm, lo cual corresponde a la etapa T2 (tumores de From 50 to 69 years old, annually
2.1 a 5cm), N0 por no mostrar metástasis axilares clínicas ni From 40 to 59 years old, every 2 years
confirmadas por patología y no tiene evidencia de De acuerdo a la NOM, se recomienda iniciar escrutinio con
enfermedad metastásica (M0), diagnosticándose como etapa mastografía a partir de los 40 años, cada 2 años, hasta los 69
clínica IIA, es decir, etapa temprana. años de edad.

33.What would be the best initial treatment for this


patient? CLINICAL CASE 8
Breast conserving surgery and axillary dissection
Hormonal therapy A 26-year-old woman presents to your practice as a new
Breast conserving surgery and sentinel node biopsy patient for routine gynecological examination. Her last
Modified radical mastectomy and axillary dissection gynecologic exam was at age 21 when she was diagnosed
with a vaginal yeast infection by the student health asks you what the relationship is between HPV and
department of her university. She reports that other than her abnormal Pap.
the yeast infection, her exam and Pap were normal. The An abnormal Pap means the cervical tissue is compromised,
patient has been sexually active since age 17 and reports 7 making it easier to contract HPV through sexual contact
partners since then. She uses condoms "about 90%" of the HPV infection is a known factor in the vast majority of
time for pregnancy and sexually transmitted disease (STD) cervical cancers (and, therefore, precancerous findings on
prevention. She has been in a monogamous relationship with Pap testing)
her current boyfriend for 2 years. She has never been HPV causes AIDS, and women with AIDS are 30% more likely
pregnant. She denies any previous diagnosis of an STD, to develop cervical cancer than women without AIDS
although she has never been tested for HIV. She is having no There is no known relationship between HPV and cervical
symptoms at this time. cancer

36.In terms of cervical cancer, what test should be El virus del papiloma humano tiene un importante papel
performed today? etiológico en el cáncer cervicouterino. Su asociación con las
Colposcopy lesiones premalignas y malignas del cérvix es igualmente
HPV testing conocida.
Pap test
Vaginal examination 39.The results of Pap testing show low-grade squamous
intraepithelial lesion. What is your next step with
El estudio recomendado para realizar escrutinio de cáncer this patient?
cervicouterino es la citología cervicovaginal. La colposcopia y Repeat Pap test in 12 months
el examen de VPH son útiles, sin embargo, no están Repeat Pap test in 6 months
recomendados como estudio inicial de escrutinio. Colposcopy
Immunization against HPV
37.In Mexico, the recommended cervical cancer
screening is: Ante el hallazgo de una citología anormal, es indicación de
Pap test from 20 to 64 years annually realizar una colposcopia, con el objetivo de revelar
Pap test from 25 to 64 years annually anormalidades que guíen la toma de biopsias para descartar
Pap test since the beginning of sexual intercourse cáncer invasor.
Pap test from 25 to 64 years, every 2 years
40.She asks about HPV immunization. According to
De acuerdo a la NOM 014 SSA2 1994 con modificaciones en national immunization schedule in Mexico, what do
2007, la edad inicial para la citología cervicovaginal es a los you recommend?
25 años, hasta los 64 años de edad. Se puede realizar cada 3 You would recommend immunization in this patient because
años solo si existen 2 citologías previas anuales sin datos de she is 26 years old
malignidad. You wouldn’t recommend immunization because in Mexico is
only indicated in 9-year-old females
38.You perform a Pap testing and the results show low- You would not recommend immunization because she has
grade squamous intraepithelial lesion (LSIL). She low-grade squamous intraepithelial lesion
There is no immunization against HPV in national 43.A b i o p s y w a s p e r f o r m e d , w i t h a c i n a r
immunization schedule in Mexico adenocarcinoma of the prostate, Gleason 3+3. PSA
12ng/mL. Which would be the best treatment for
En México, de acuerdo a la NOM 036 SSA2 2012 se contempla this patient?
vacunación a los 9 años de edad. Radical prostatectomy
External beam radiotherapy
CLINICAL CASE 9 Watchful waiting
Transurethral prostatic resection
A previously healthy 65-year-old male presents to your
practice with a case of 3 months with nocturia, difficulty En términos generales, la enfermedad clínicamente
starting and maintaining a steady stream of urine, and confinada a la próstata se trata primero con cirugía en
dysuria. No other symptoms. Digital rectal examination with pacientes con expectativa de vida mayor a 10 años y buenos
suspicious lump in right side of the prostate. No other candidatos quirúrgicos. Se puede considerar observación en
physical findings. un tumor bien o moderadamente diferenciado con esperanza
de vida menor a 10 años. La radioterapia primaria es una
41.What will be your next step with this patient? buena alternativa en pacientes que rechazan la cirugía y en
Urine examination and urine culture los que no son buenos candidatos quirúrgicos. La RTUP está
Start treatment with tamsulosin indicada solo en casos que no pueda realizarse cirugía y
Referral to Urology for transurethral prostatic resection requiera procedimiento paliativo de obstrucción.
Perform a PSA test and transrectal ultrasound with biopsy
44.Which is the most frequent localization of prostate
Se trata de un hombre > 60 años con síntomas irritativos y cancer?
obstructivos de próstata, además de una masa sospechosa al Central zone
tacto rectal. El abordaje diagnóstico se basa en realizar APE Peripheral zone
y ultrasonido transrectal, el cual permite identificar lesiones Transitional zone
sospechosas para guiar la toma de biopsia. Prostatic urethra

42.Which one is your most likely diagnosis? Entre 70-85% de los carcinomas ocurren en la zona
Benign prostatic hyperplasia periférica, 10-15% en la zona transicional y 10% en la zona
Prostate cancer central, hasta 85% son multifocales.
Prostate abscess
Chronic prostatitis 45.Which one is not a risk factor for prostate cancer?
Black ethnicity
De acuerdo al hallazgo de la exploración digital rectal, la Vitamin E consumption
sospecha clínica es de cáncer de próstata. La hiperplasia Alcohol consumption
prostática benigna presenta también síntomas irritativos y Cigarrete smoking
obstructivos, sin embargo, el tacto rectal se presenta con
aumento de volumen sin mostrar lesiones predominantes. Los factores de riesgo identificados son edad, raza, historia
familiar, tabaquismo, alcoholismo y actividad física. Algunos
estudios demuestran cierta acción protectora en sustancias Radiotherapy
retinoides de vegetales o frutas, vitamina E, selenio y Tyrosine-kinase inhibitors
vitamina D. Renal embolization because she has gross hematuria

CLINICAL CASE 10 Si el tumor está localizado debe realizarse nefrectomía


radical, que sigue siendo el único tratamiento efectivo para
A 65-year-old female arrives to emergency department with el cáncer localizado. Se considera un tumor
gross hematuria. She had hematuria since 3 months ago, quimiorresistente y radiorresistente.
with right-side flank pain and weight loss. Physical
examination with pallor, and you detect a palpable mass in 49.Which one is not a risk factor for renal cancer?
abdomen. Von Hippel Lindau syndrome
Cigarette smoking
46.Which one is your most likely diagnosis? Chronic renal disease
Urinary tract infection Chronic urinary infections
Bladder cancer
Renal cancer Dentro de los factores de riesgo conocidos se encuentra la
Renal glomerulonephritis} dieta, la falla renal crónica, tabaquismo, obesidad y algunos
factores relacionados con cáncer renal hereditario como
Se trata de un hombre de 65 años con hematuria, dolor en síndrome de von Hippel-Lindau y carcinoma papilar de
flanco y masa palpable, que juntos se conocen como la células renales.
triada típica de cáncer renal, presente hasta en el 10% de los
pacientes. Otras características son fiebre o pérdida de peso. 50.Which one is the most frequent histological type of
renal cancer?
47.What would you do to confirm your diagnosis? Sarcoma
Cystoscopy Clear-cell carcinoma
Abdominal CT Transitional carcinoma
Renal biopsy Cromophobe carcinoma
Renal ultrasound
La clasificación de la UICC describe 5 subtipos. Por orden de
La TC confirma el diagnóstico y valora la morfología y frecuencia son: carcinoma de células claras (60-75%), papilar
función del riñón contralateral, además de la extensión (15%), cromófobo (5%), no clasificables (5%).
extrarrenal y la afección vascular y ganglionar.

48.You find an 8-cm renal mass with no invasion to


adjacent structures and with no thrombi, chest x-
Ray without metastases, and laboratory test only
with anemia, liver function tests normal. The best
treatment for this patient is:
Radical nephrectomy

También podría gustarte